MCQ Questions
MCQ Questions
1. A 21-year-old man comes to the clinic because he has become increasingly short of breath and has had a cough for the past week. He
appears dyspneic and has a temperature of 38.3EC (101.0EF). On physical examination he has bibasilar rales and generalized lymphadenopathy
(1 to 2 cm). Rectal examination shows multiple perianal contusions and a small amount of blood oozing from the anal orifice. Chest x-ray film
shows bilateral patchy alveolar infiltrates. The most appropriate course of action is to order blood tests and to do which of the following?
(A) Admit him to the hospital and begin administration of trimethoprim-sulfamethoxazole, intravenously
(B) Admit him to the hospital and begin administration of penicillin and gentamicin, intravenously
(C) Begin administration of erythromycin, orally, and see him again the next day
(D) Prescribe isoniazid and rifampin, orally
(E) Recommend aspirin, fluids and rest at home
2. A 17-year-old white girl returns to the health center for a family planning follow-up visit. She gave birth to a healthy baby girl 8 months ago
and does not want to become pregnant again. She is monogamous with the father of her baby but worries that he is not monogamous with her.
They live together and are not married. She is taking an oral contraceptive and "sometimes" uses condoms. She says that she has great trouble
remembering to take her pills and wants to discuss other contraceptive options. In addressing this issue, which of the following is the most
appropriate next step?
(A) Advise her to continue taking an oral contraceptive because it is one of the most effective methods of birth control
(B) Advise her that an intrauterine device would be a good contraceptive choice for her
(C) Discuss implantable or injection long-term progestational contraceptive agents
(D) Discuss the option of tubal ligation
(E) Fit the patient with a diaphragm and instruct her on proper use
3. A 19-year-old African-American college student comes to the student health center because of marked fatigue. Temperature is 38.3°C .
Physical examination shows striking pallor of skin, nail beds and conjunctivae. There are petechial hemorrhages in the skin of his legs. A soft,
blowing systolic murmur is present over the precordium. No other abnormalities are present. Which of the following is the most appropriate
study at this time?
4. A 12-year-old girl is referred to the clinic by the school nurse for evaluation of scoliosis. The girl's scoliosis was detected during a routine
screening examination at the school, and it appears to be mild (curve less than 10 degrees). She is athletic and is otherwise in good health.
During the physical examination, particular attention should be given to which of the following?
5. A 10-month-old infant is brought to the health center by his mother because of eight watery bowel movements during the past 24 hours. On
physical examination the infant is lethargic and somnolent. The mucous membranes are dry and the skin turgor is poor. The anterior fontanel is
sunken. At his last visit 2 weeks ago, he weighed 10 kg ; today his weight is 9 kg . His temperature is 37.2 , pulse is 170/min and blood pressure
is 100/60 mm Hg. Which of the following is the most appropriate next step in management of this infant?
A 45-year-old woman is brought to the health center by her husband because of nausea, confusion, chills, fever, flank pain and cloudy urine.
She has a history of insulin-dependent diabetes mellitus, poorly controlled hypertension and recurrent urinary tract infections. Her vital signs
are: Temperature 40.0°C Pulse 120/min Respirations 24/min Blood pressure 110/70 mm Hg Funduscopic examination shows diabetic
retinopathy, which is unchanged from the previous examination. Neck is supple. Lungs are clear to auscultation and percussion. Examination of
the abdomen is normal. Marked pain is present at the right costovertebral angle. Several hemorrhagic bullous lesions are noted on the
extremities.
6. Without prompt and aggressive treatment, this patient is most likely to develop which of the following?
7. If this patient were to develop anuria, which of the following would be the most likely cause?
8. A 23-year-old registered nurse comes to the employee health clinic because she says, "I'm too tired to work." She has had increasing fatigue,
malaise and anorexia during the past several days. Laboratory studies show: Serum Blood ALT 1160 U/L PT 13 sec Bilirubin 1.8 mg/dL HBsAg
Positive She is instructed to rest at home and return in 3 days if no new symptoms develop. Two days after the visit she calls to say that she has
now developed an urticarial rash and swelling of the joints of her fingers. At this time which of the following is the most correct statement
about her condition?
(A) The arthritis and rash are the result of an associated immune complex disorder
(B) The arthritis and rash are unrelated to her liver disease
(C) It is unlikely that her blood is infectious
(D) She has a 50% risk for developing chronic liver disease
(E) She should be given hepatitis B immune globulin
9. A 47-year-old Italian-American man comes to the office for the first time for routine medical care. He has been referred to you by his
psychiatrist who has informed you that the patient has paranoid personality disorder. The patient has no other medical problems at this time.
He is unmarried, lives alone and has no close friends, but he occasionally attends family gatherings. He functions well working alone in a
technical position in an engineering firm. Which of the following is the best way to structure the physician-patient relationship with this
patient?
(A) Avoid giving him excessive details about possible, but infrequent, side effects and complications in order to avoid triggering his paranoia
(B) Explain the rationale for any diagnostic procedures and treatment regimens in some detail, adopting a professional, but not overly friendly
stance
(C) Go out of your way to be warm and friendly so that he can develop trust in you
(D) Have his psychiatrist, with whom he has been working for several years, take the lead in presenting medical treatment options
(E) Try to communicate with his family or medical personnel when he is not present in order to overcome his withholding information because
of distrust
10. A 12-month-old boy is brought to the office by his mother for his routine health check-up. She informs you that she has just been
diagnosed with hypercholesterolemia. Her fasting serum total cholesterol concentration was 260 mg/dL and her LDL-cholesterol
concentration was 130 mg/dL (rec<129 mg/dL). She is unaware of a family history of coronary artery disease because she was
raised by her godmother when her parents died in their early 30s in a motor vehicle accident. A special diet has been recommended
for her; however, she is very concerned about the risk of hypercholesterolemia for her son. Which of the following is the most
appropriate management at this time?
(A) Ask her to reduce the child's fat intake and give him skim milk instead of whole milk
(B) Do nothing until the child is 2 years of age
(C) Have the child return for a fasting lipoprotein analysis
(D) Obtain a random serum total cholesterol concentration for the child today
(E) Refer the child to a lipid specialist
11. A 75-year-old white man comes to the office because of increasing shortness of breath on exertion for the past 2 to 3 months. He has a
history of hypertension for which he takes hydrochlorothiazide. On physical examination his pulse is 80/min. There is an apical
lift displaced to the left and a harsh diastolic murmur at the base. Further physical examination is most likely to show which of the following?
12. A 33-year-old white woman asks you for a third opinion because two other physicians have been unsuccessful in alleviating her multiple
symptoms. She complains of numbness in her face and a pain deep behind her left eye. She describes weakness in her upper extremities and a
"clumsy right hand." She says she intermittently wets her pants. She relates her history in an emotional fashion and emphasizes that the
symptoms are all worse when the weather is hot. Physical examination discloses increased deep tendon reflexes in both biceps and triceps.
There is temporal pallor and partial atrophy of the left optic nerve head. Which of the following studies will most likely confirm the diagnosis?
13.
The lesion shown is of a 1-year-old girl who is brought to the office by her parents because they want a second opinion regarding the growth on
her leg. The mother notes that it was small and flat at birth and has grown thicker in the past 6 months. The mother says, "My usual physician
seems unconcerned, but my mother-in-law says it looks awful and that something should be done." The child is otherwise healthy. At this time
it is most appropriate to tell the parents that the best cosmetic result will occur with which of the following?
14. A 15-year-old boy comes to the office for a sports participation physical examination. He has been playing in a summer basketball league
and now wants to try out for the high school team. His last physical examination was 2 years ago and, according to him, he has been healthy
except for a cold 2 weeks ago. Before you begin the physical examination, the nurse informs you that his routine urinalysis shows: Color Tea-
colored/dark WBC 7/hpf Specific gravity 1.030 RBC >100/hpf, a few red cell casts pH 5.5 Bacteria Negative Protein 2+ Glucose Negative Ketones
Negative These laboratory results are most indicative of which of the following?
(A) Cystitis
(B) Glomerulonephritis
(C) Nephrotic syndrome
(D) Pyelonephritis
(E) Renal calculi
15. A 16-year-old Anglo-American girl comes to the office because she has missed a menstrual period and a home pregnancy test was positive.
Bimanual examination discloses an enlarged uterus, and a urine pregnancy test is positive. She is estimated to be approximately 6 to 8 weeks
pregnant. You have been the family's physician since she was born. She asks that you please not tell her parents. Which of the following is the
most appropriate immediate response?
(A) Attempt to persuade her to share the information with at least one adult member of her family
(B) Explain that by law you are unable to maintain confidentiality
(C) Explore with her the reasons for her not feeling comfortable sharing the information with her parents
(D) Point out that she will not be able to hide her pregnancy for very long
(E) Reassure her, but notify her mother after she leaves
16. During a routine physical examination of a 2-week-old Jewish neonate, a grade 3/6 early systolic murmur is heard. The mother reports no
signs or symptoms of illness in the baby. The neonate is otherwise asymptomatic. Physical examination at birth was normal. Which of the
following is the most appropriate next step?
17. A 29-year-old Italian-American woman comes to the office for her first prenatal visit. Her last menstrual period was 16 weeks ago. This is her
first pregnancy; her family history is unremarkable. She has heard that people of Mediterranean ancestry are at risk for
carrying a gene for ?-thalassemia. She asks to be tested for this. Which of the following is the most appropriate initial diagnostic study?
(A) Complete blood count with red cell indices
(B) Hemoglobin electrophoresis
(C) Red cell osmotic fragility test
(D) Restriction-fragment length polymorphism (RFLP) analysis of her ?-globin gene
(E) Reticulocyte count
18. A 22-year-old man returns to the office for a follow-up visit 4 weeks after being diagnosed with mononucleosis. He is a college football
player and the football season is just beginning. At his initial visit, he had a sore throat, nausea and a temperature of 39.4EC . Physical
examination at that time showed pharyngitis, submandibular and posterior cervical lymphadenitis, and
splenomegaly. Complete blood count (CBC) done at that time showed a leukocyte count of 18,000/mm3 with an elevated number of
monocytes, and a Monospot® test was positive. The course of his illness has been uncomplicated, but today he still complains of some residual
fatigue. Before you examine him, he asks you, "When can I play again?" Which of the following is the most appropriate response to the patient
regarding when he may return to contact sports?
(A) Next season
(B) When a Monospot® test is negative
(C) When he is no longer symptomatic
(D) When his CBC is normal
(E) When his physical examination is normal
Items 19-20
A 34-year-old man comes to the office because he has experienced 6 weeks of gradually increasing fevers, dry cough and shortness of breath.
Records show he has lost 5.4 kg since his last routine visit 6 months ago. Today, on physical examination, there are several large, nontender,
anterior and posterior cervical lymph nodes. His medical history includes infectious mononucleosis, hepatitis B,impetigo, prostatitis and tinea
versicolor.
19. Which of the following factors in his history is most suggestive of pneumonia related to a sexually transmitted disease?
(A) Hepatitis B
(B) Impetigo
(C) Infectious mononucleosis
(D) Prostatitis
(E) Tinea versicolor
20. Which of the following is the most appropriate question to ask this patient when inquiring about his sexual orientation?
(A) "Have you been sexually active with men, women, or both?"
(B) "Have you ever had homosexual relations?"
(C) "How many girlfriends have you had?"
(D) "How many times have you had sex with other men?"
(E) "What are your views about homosexuality?"
21. A male colleague asks you to write a prescription for a narcotic analgesic for one of his female patients. You have noticed that this patient
frequently has been coming by the office to see your colleague, and that several of the visits have been marked "No Charge." When you ask
your colleague why he cannot write the prescription himself, he seems defensive and says, "Because I don't want anybody to get the wrong
idea." Which of the following is the most appropriate response to your colleague?
(A) "It sounds like there is more to this story than you are telling me; maybe we should talk about it."
(B) "I wish I could help you, but I never prescribe that medication for a patient unless I have seen the patient myself."
(C) "I will do this for you once, but I will need to see your patient in the office before I can write another prescription."
(D) "Maybe I should see your patient in the office myself, and then decide if she needs the medication."
(E) "You have seemed a little nervous lately. You aren't getting in over your head, are you?"
22. A 26-year-old man whom you have treated with alprazolam for anxiety contacts you via the answering service concerning his 20-year-old
sister. You have seen his sister in your practice for routine family planning and preventive care. He speaks excitedly, and says, "My sister is just
lying on the floor moaning. She fell on the floor while we were eating dinner. There wasn't any reason for it,except the room was very hot from
the cooking." According to her brother, she slowly slumped over onto her husband while they were sitting at the dinner table. The family then
eased her onto the floor. She denies any pain and speaks in complete sentences. Her
brother sees no rash and no evidence of bruising. The patient looks at him when she is talking, but she intermittently turns away, moving her
head from side to side and moaning and crying. He also tells you that his sister and her husband kept an appointment with their clinical
psychologist today for marriage counseling. According to the information available, which of the following is the most likely cause of this
behavior?
23. A 22-year-old woman returns to the office for a follow-up visit. She was last seen 2 weeks ago because of a facial rash that worsens with sun
exposure. Today she says she has joint pain. She says, "It just hurts everywhere. Sometimes my knees hurt, and sometimes it's my elbows or my
ankles. And I have been feeling tired all the time. I just don't have any energy." She denies any joint swelling. An office urinalysis shows 3+
protein. You consider the diagnosis of systemic lupus erythematosus and plan additional diagnostic testing. If she has systemic lupus
erythematosus, studies are most likely to show which of the following?
Items 24-25
A 59-year-old woman comes to the office because of pain in the left lower quadrant of her abdomen for the past 48 hours. The pain is
associated with tenesmus, loss of appetite and a sensation of being febrile. She has a history of long-standing irritable bowel symptoms and
has not taken any pharmacotherapy. Vital signs are: temperature 37.3EC (99.2EF), pulse 84/min and regular and respirations 12/min. On
physical examination the abdomen is tender in the left lower quadrant; there is no rebound tenderness and bowel sounds are normal.
Leukocyte count is 10,200/mm3 with 71% segmented neutrophils and 3% band forms. The patient is sent home on a liquid diet and tetracycline
therapy. Three days later she returns because the pain has becom worse and she is now having chills.
25. Following the patient's recovery from the acute phase of this illness, which of the following is the most appropriate long-term
management?
(A) Annual endoscopic examination for neoplasia
(B) High-fiber diet
(C) Low-residue diet
(D) Reassurance only
(E) Use of prophylactic antibiotic therapy for 1 week each month
END OF SET
26.
A husband and wife, ages 29 years and 34 years, respectively, come to the office for advice regarding their risk for having hearing impaired
children. They are both hearing-impaired and require hearing aids. Their hearing loss is sensorineural and is not associated with any other
health problems. The wife tells you, "We have both learned to live with this disability, but we want to take it into account before we decide to
have children." Their pedigree is shown with the patients identified as II2 and II3. Which of the following is the most appropriate advice to the
couple?
(A) Accurate risk estimation is impossible without further evaluation
(B) Because they are hearing-impaired, all their children will be hearing-impaired
(C) It is unlikely that they will have hearing-impaired children
(D) Only their male children will be hearing-impaired
(E) They can have amniocentesis during pregnancy to test whether the infant will be hearing-impaired
Items 27-29
An 18-year-old white high school student comes to the office in late August because of a stuffy nose for 1 week. He reminds you that he has had
severe hay fever in the fall for the past 10 years. Review of his chart shows that he has positive skin tests to ragweed, dust and dust mites, and
he is receiving maintenance immunotherapy with extracts of these antigens. He also takes over-the-counter antihistamines for symptomatic
relief. This regimen has not provided relief so far this season. Physical examination is normal, apart from clear rhinorrhoea. He is afebrile, and
there is normal transillumination of the frontal and maxillary sinuses. In reviewing his medical records you note that smears of his nasal mucus
contained large numbers of eosinophils. There is no record of any extensive immunologic work-up. You tell him that he now has either an early
viral upper respiratory tract infection or the beginnings of his seasonal allergic rhinitis. You suggest
that he use a corticosteroid nasal spray plus his usual antihistamines as needed. The appropriate treatment is undertaken, but 4 days later he
returns because of a toothache and fever. On physical examination he has right facial fullness and pain below his eye when he leans forward.
There is tenderness in the region of the upper premolar and molar teeth on the right side. His temperature is 38.7 orally. He has bloody, thick,
green mucus coming from his right nostril. The remainder of his physical examination is normal. You suspect maxillary sinusitis on the basis of
the clinical findings.
27. Before beginning antibiotic treatment in this patient, it is necessary to first do which of the following?
(A) Confirm the diagnosis with CT films of the sinus
(B) Confirm the diagnosis with plain x-ray films of the sinus
(C) Confirm the diagnosis with transillumination of the sinus
(D) Request consultation with a dentist
(E) No additional steps are necessary
28. The appropriate steps are taken. In prescribing antibiotic therapy for this patient, it is most important to remember which of the following?
(A) Antibiotics are ineffective unless there is a concomitant surgical drainage procedure
(B) He is likely to be allergic to penicillin
(C) He needs to take medication for more than 7 days
(D) Only bactericidal antibiotics are effective
(E) Ordinarily at least two antibiotics are given
He is treated and initially feels much better. However, soon thereafter he developed a headache, right ear pain and painful stiff neck; he spikes
a temperature to 39.3EC (102.7EF), orally. Extraocular movements are normal.
29. Which of the following is the most likely explanation for these new symptoms?
(A) Allergic reaction to the antibiotic(s)
(B) Associated meningeal inflammation or infection
(C) Development of cavernous sinus thrombosis
(D) Direct spread of infection from the maxillary to the mastoid sinus
(E) Obstruction of the orifice of the maxillary sinus with a mucous plug
END OF SET
30. A 50-year-old African-American man with severe chronic obstructive pulmonary disease returns to the office following a recent evaluation
for possible lung transplantation in another city. He says he had been considered a suitable candidate, in all respects, but was rejected by the
transplant program when a random urine test was positive for a nicotine metabolite. He had previously told you that he had stopped smoking 3
years ago. He stands by this and is at a loss to explain the positive urine test. He wants to know what he should do now. Which of the following
is the most appropriate next step?
(A) Advise him again to stop smoking and refer him to another transplant program
(B) Advise him that the transplant program cannot turn him down on this basis, according to the Americans with Disabilities Act
(C) Contact the transplant program to learn their reasons for turning him down
(D) Explain to the patient that transplantation is out of the question as a result of what has occurred
(E) Write to the transplant program and insist that they give him another opportunity
Items 31-33
A 43-year-old teacher, who is the mother of three children, comes to the office for evaluation of high blood pressure. An elevated blood
pressure was first detected 1 week ago during a routine screening at health fair in a local shopping mall. On examination today her blood
pressure is 145/95 mm Hg. 31. Which of the following is the most appropriate next step?
(A) Advise her to monitor her blood pressure twice a day at home and return to the office in 6 months
(B) Ask her to return for reexamination after her next menstrual cycle
(C) Ask her to return for reexamination in 2 weeks
(D) Ask her to return for reexamination in 4 months
(E) Measure her blood pressure after she exercises for 5 minutes
32. Hypertension is confirmed. She is started on a low-sodium diet and an antihypertensive medication. In addition, she should be advised that
she should do which of the following?
(A) Maintain ideal body weight
(B) Restrict her physical activities
(C) Seek less stressful employment
(D) Take 300 mg of aspirin, daily
(E) Use a combination oral contraceptive
33. On physical examination 1 year later she is asymptomatic and her blood pressure is 140/85 mm Hg. She is continuing to take the prescribed
antihypertensive medication. Which of the following is the most appropriate management at this time?
(A) Continue the antihypertensive therapy
(B) Discontinue the antihypertensive therapy
(C) Order a chest x-ray film and electrocardiography
(D) Order complete blood count, serum electrolyte concentrations and liver chemistry profile
(E) Reduce the dosage of the antihypertensive medication by 50%
END OF SET
34. A 24-year-old woman comes to the office because of concerns about sexual function. The patient recently has remarried after being
divorced from an abusive partner. She is currently unable to complete intercourse with her new husband due to intense vaginal pain on
attempts at penetration. She was able to have intercourse successfully early in her first marriage. She is orgasmic with other stimulation but
cannot tolerate digital or other penetration. Vital signs are: temperature 37.0, pulse 72/min, respirations 18/min and blood pressure 118/64
mm Hg. Physical examination is normal; however, the patient is unable to tolerate the speculum examination because of pain. Which of the
following is the most likely mechanism for this condition?
(A) Change in vaginal flora because of a new partner
(B) Conversion disorder
(C) Inadequate lubrication
(D) Inflammation of vestibular glands
(E) Vaginal muscle spasm
35. A 67-year-old woman who is a regular patient calls the office because she has developed severe muscle weakness, muscle cramps and
polyuria. She began treatment 6 weeks ago with 50 mg of chlorthalidone daily for mild-to-moderate essential hypertension. The most likely
explanation for her symptoms is the development of which of the following?
(A) Diabetes mellitus
(B) Hypokalemia
(C) Hypomagnesemia
(D) Hyponatremia
(E) Metabolic acidosis
36. A recent, large clinical trial assessed the effect of digoxin on morbidity and mortality in patients with chronic congestive heart failure (CHF).
In the trial, more than 7000 patients with left ventricular ejection fractions of 0.45 or less were randomly assigned to receive digoxin or placebo.
All patients also were treated with diuretics and an angiotensin-converting enzyme (ACE) inhibitor. The patients were observed for an average
of 37 months. During the clinical trial, 34.8% of patients treated with digoxin and 35.1% of patients treated with placebo died (relative
risk=0.99; confidence interval 95%=0.91 to 1.07, p=0.80). The best interpretation of
these data is that in combination with a diuretic and ACE inhibitor, digoxin therapy results in which of the following?
(A) A beneficial effect on mortality rates
(B) A beneficial effect on patients with CHF
(C) No conclusive evidence of effect because of the limited power of the study
(D) No effect on CHF
(E) No effect on mortality rates
37. A 6-year-old Mexican-American boy is brought to the office by his mother because of the gradual onset of abdominal pain. The visit was
prompted by the child's first grade teacher who sent him home from school because he appeared ill. The boy has been above the
recommended weight for his height at every visit since he became your patient 5 years ago. On physical examination today the child does not
appear to be in acute distress. Abdominal examination discloses normal bowel sounds and no tenderness, and is otherwise inconclusive. Which
of the following is the most appropriate next step?
(A) Ask the mother about problems separately from her son
(B) Call the school to check on details of the onset of pain
(C) Do rectal examination
(D) Obtain serum liver chemistry profile
(E) Order supine x-ray film of the abdomen
38. A 44-year-old general surgeon is convicted of narcotic abuse. He successfully completes a 3-month drug rehabilitation program. Following
this treatment, his privilege to practice medicine will be determined by which of the following?
(A) American Board of Surgery
(B) Credentials committee of his hospital
(C) State medical board
(D) US Drug Enforcement Agency
(E) He will not be permitted to practice
39. A 76-year-old woman returns to the office because of aching and weakness in her arms to the point where she cannot lift her arm to brush
her hair. Physical examination shows no muscle tenderness or other evidence of joint disease in both arms. The aching improves when she
takes the prescribed nonsteroidal anti-inflammatory drug (NSAID). She also describes tenderness over the right parietal area of her scalp.
Physical examination of the scalp shows no lesions. Which of the following is the most appropriate next step?
(A) Increase the dose of the NSAID
(B) Order determination of erythrocyte sedimentation rate
(C) Order determination of serum rheumatoid factor
(D) Order x-ray films of the cervical spine
(E) Refer her for psychiatric counseling
40. A 34-year-old laborer comes to the office because of a 2-kg weight loss and an increased appetite. He has diabetes mellitus and has been
taking insulin in divided doses. He says that home monitoring of his serum glucose concentration has shown values from 280 mg/dL to 320
mg/dL. Which of the following is the most appropriate management?
(A) Add metformin
(B) Change to another type of insulin
(C) Increase his caloric intake
(D) Increase his insulin dose
(E) Redistribute his caloric intake
41. An 84-year-old woman is brought to the office by her daughter, who is your patient. The mother has just moved in with the family because
she can no longer care for herself due to progressive, long-standing dementia. The daughter hopes you will help take care of
her mother. On physical examination the mother does not respond to your words or to the physical examination. You notice that she smells of
urine. On examination of the pelvis there is a diffuse erythematous rash extending over the perineum and the medial
thighs bilaterally. You suspect the rash relates to urinary incontinence. The daughter is present at the mother's examination. Which of the
following is the most appropriate statement to the daughter?
(A) "How long have you left your mother in this condition?"
(B) "His is a rash caused by urine. My nurse will insert a Foley catheter."
(C) "This rash should respond to cleansing with mild soap and drying with a clean towel three times a day."
(D) "You should take better care of your mother."
(E) "Your mother needs to wear diapers."
42. You are discussing treatment options with a 75-year-old woman with chronic renal failure. Her medical history also includes
diabetes mellitus, hypertension, hypercholesterolemia, and a left above-the-knee amputation secondary to severe peripheral vascular
disease. She lives with her dog in a one-bedroom apartment in the suburbs and is dependent upon a wheelchair for ambulation. She
has a fifth-grade education. Her support system includes a home health aide, several friends who live 5 to 6 miles from her
apartment and Meals-on-Wheels. Which of the following is the most appropriate advice to give this patient regarding long-term
management?
(A) Dialysis is not an appropriate option because of her age and medical disabilities
(B) Home dialysis would permit the greatest independence and convenience
(C) Peritoneal dialysis is not an option because of her diabetes mellitus
(D) She must name a health care surrogate to make decisions regarding her care
(E) You will arrange for dialysis at a center near her home if she can arrange transportation
Items 43-44
A 34-year-old, white, married business executive who is a patient in your practice comes to the office because of low back pain. After you
enter the examining room and establish rapport, he appears embarrassed and says that back pain is not why he is here. Upon further
questioning, he admits to a sexual encounter on a recent business trip and says he now has a greenish urethral discharge. Tests for
gonorrhea are positive. He admits to having exposed his wife since returning from the trip. He does not want his wife told about this.
43. Because his wife is also a patient in your practice, which of the following is the most ethical approach?
(A) Report the case to the state health department and let them handle contacting his wife
(B) Request that he inform his wife of her exposure and the need for treatment and offer to assist as necessary
(C) Suggest that he bring his wife to the office tomorrow so that you can explain the situation and the need for treatment
for both of them
(D) Telephone his wife after he leaves, inform her of the problem and request that she come to the office for treatment
(E) Treat him now and treat his wife when she comes to the office for her next annual Pap smear 2 weeks from now
44. The patient decides he will personally tell his wife that night. He calls you the next day stating that his wife is angry and distraught,
and she is contemplating divorce. She is so upset she cannot even come to the office for treatment. He wants to know what to do
now. Which of the following is the most appropriate next step?
(A) Call the wife to ask her to come to the office to be treated and to talk about her reaction to this situation
(B) Have a member of your nursing staff call the wife to stress the importance of coming to the office for treatment
(C) Make a house call to treat her and assess the situation
(D) Refer them to a marriage counselor and ask her to come to the office next week for treatment
(E) Refer them to a psychiatrist skilled in crisis intervention and give her an antidepressant medication until the next
available appointment in 3 weeks
END OF SET
45. A 28-year-old man comes to the office because he is concerned about his relationship with his 9-year-old son. He is a single father
who has full custody of his son. The father says that they always have been very close, but recently he has not had much energy to
spend quality time with his son. He does not understand why he has had this change in behavior. He says he feels well, and to his
knowledge, is healthy. He says that he works daily in a job he likes and has had no occupational problems. History and physical
examination lead you to suspect substance abuse. Abuse of which of the following substances would most likely explain his
behavior?
(A) Amphetamine
(B) Cocaine
(C) Marijuana
(D) Phencyclidine
(E) Psilocybin
52
46. A 28-year-old woman who is 20 weeks pregnant with her third child comes to the office for a routine prenatal visit. She has two
healthy children, ages 7 and 5 years. Her previous pregnancies were uncomplicated except for a cesarean delivery of the first child.
She began using cocaine soon after the birth of her second child, and she has been in and out of drug treatment programs for the
past 3 years. She has used crack cocaine on and off throughout this pregnancy. You have encouraged her to seek help; however, she
has made no attempt to abstain from using cocaine and she refuses to commit herself to another drug treatment program. In your
attempt to persuade this patient to stop using cocaine, you advise her that if she continues to use cocaine during the pregnancy, she
increases her risk for which of the following?
(A) Chorioamnionitis
(B) Gestational diabetes
(C) Placental abruption
(D) Placenta previa
(E) Preeclampsia
47. A 22-year-old college student calls your evening answering service because of an itchy, spreading rash over her right wrist and
lower arm for the past 2 days. You return her call, and she reports that she just returned from a weekend camping trip in the
mountains. She denies any fever, chest pain, dyspnea or red streaks up her arm. She states, "I'm feeling miserable and the itch is
killing me. I don't know if I was bitten by some insect or a tick. What should I do?" In counseling the patient over the telephone,
which of the following is the most appropriate response?
(A) "From what you describe I think you should go directly to a dermatologist. Please come to my office in the morning for
a referral."
(B) "Go to your pharmacist and ask for an over-the-counter antihistamine and Burow's solution, which you should apply
with wet compresses, and please come to my office in the morning."
(C) "I will call in antibiotics to the pharmacy so that you can begin treatment tonight. I would like to see you in my office
tomorrow."
(D) "Unfortunately I cannot prescribe any treatment without first examining you. Please call my office in the morning for
an appointment."
(E) "You should go to the nearest emergency department immediately for treatment and come to my office tomorrow
morning for follow-up."
48. During a sports physical examination, a 13-year-old boy expresses concern about his height. He was in the 10th percentile for
height throughout childhood and now is just above the 5th percentile. His sexual maturation rating is Tanner stage 2 for both
genitalia and pubic hair. He should be told which of the following?
(A) He is near his final height
(B) He probably has a slight deficiency of growth hormone
(C) He should increase his daily caloric intake
(D) He will grow several more inches
(E) He will need testosterone injections for several months
NOTE: THIS IS THE END OF THE OFFICE/HEALTH CENTER BLOCK.
ANY REMAINING TIME MAY BE USED TO CHECK ITEMS IN THIS BLOCK.
53
Block 2: Office/Health Center
Items 49-96; Time - 60 minutes
You see patients in two locations: your office suite, which is adjacent to a hospital, and at a community-based health center.
Your office practice is in a primary care generalist group. Patients are seen for routine and urgent care at the office and
health center. Most of the patients you see are from your own practice, although occasionally you will see a patient cared for
by one of your associates and reference may be made to the patient's medical records. Known patients may be managed by
telephone, and you may have to respond to questions about information appearing in the public media, which will require
interpretation of the medical literature. The laboratory and radiology departments have a full range of services available.
ALL ITEMS REQUIRE SELECTION OF ONE BEST CHOICE.
49. A 14-year-old boy is brought to the walk-in clinic by his father late on Saturday afternoon because his left ear is swollen and
painful. The boy's ear has been black and blue since he injured it in a wrestling match 3 days ago. Symptoms have increased
significantly following a repeat injury 3 hours ago. On physical examination, his left ear is markedly swollen and tender to
palpation. Which of the following is the most appropriate next step?
(A) Reassure him and start aspirin therapy
(B) Reassure him and start codeine therapy
(C) Recommend that he apply cold packs to the ear for the next 12 hours
(D) Recommend that he apply hot packs to the ear for the next 12 hours
(E) Refer him to a surgeon for immediate drainage of the lesion
Items 50-51
A 6-week-old infant is brought to the health center by his mother because of projectile vomiting. You have not seen this infant before
and the mother says he has not been seen by a physician since birth. Weight at birth was 3550 g and physical
examination was normal. Examination today is normal except for weight, now 4000 g , and slight dehydration.
50. Which of the following is the most appropriate management for this infant?
(A) Elevate the head of his crib to relieve gastroesophageal reflux
(B) Order supine and decubitus x-ray films of the abdomen
(C) Order ultrasonography of the pylorus
(D) Order an upper gastrointestinal barium study
(E) Place him on a clear pediatric electrolyte solution
51. The appropriate diagnostic test is done and is equivocal. Which of the following imaging modalities necessary to help
diagnose this infant's illness is most operator-dependent?
(A) CT scan
(B) Radionuclide gastric emptying time studies
(C) Routine x-ray films
(D) Ultrasonography
(E) Upper and lower gastrointestinal barium studies
END OF SET
54
52. A 75-year-old woman comes to the clinic because she has band-like burning pain in the right upper quadrant extending from
the epigastrium around to the midline of the back. On physical examination, there is no abdominal tenderness. Findings on
ultrasonography of the gallbladder are normal; serum amylase concentration is normal. Which of the following is the most
likely diagnosis?
(A) Acalculous cholecystitis
(B) Chronic relapsing pancreatitis
(C) Diverticulitis of the cecum
(D) Herpes zoster
(E) Penetrating duodenal ulcer
53. A 2-year-old boy who has recently become one of your patients is brought to the clinic by his mother for a follow-up visit of a
chromosome analysis done 1 month ago. This child has minor dysmorphic features, and growth and developmental delay.
Chromosome analysis showed a small unbalanced chromosome translocation, with extra chromosomal material at the tip of
chromosome 3. The cytogenetics laboratory requested blood samples from both parents for follow-up studies. The parents are
divorced, and the mother has custody of the child. The relationship between the parents is hostile. The mother has been tested
and has normal chromosomes without evidence of translocation. At today's visit, she reacts angrily when the issue of
contacting the child's father for testing is raised. She states that he abandoned them, and that he has no interest in his child.
She refuses to cooperate in contacting the father, who could be a translocation carrier. You do not know the father, but an
office worker told you that he lives in a nearby town. The mother says that he is living with a new girlfriend. Which of the
following is the most appropriate next step?
(A) Attempt to identify the father's physician and work with that physician to obtain chromosome studies on the
father
(B) Contact the father by telephone and arrange for him to give a blood sample at a local hospital
(C) Document your attempts to work with the mother but proceed no further, since you have no physician-patient
relationship with the father
(D) Help the mother deal with her anger and educate her regarding the potential benefit to her son and others if the
father's chromosome studies are done
(E) Send the father a letter (expressing few details about the patient) and suggest that he contact your office for an
appointment and further discussion of his child
54. An 18-month-old white infant is brought to the clinic because of pallor and irritability. Her mother says the infant's diet
consists almost exclusively of whole milk, approximately 40 oz per day. On physical examination, the infant has a pulse of
160/min, respirations of 50/min, and normal heart sounds with a grade 2/6 systolic ejection murmur. Liver is palpable 3 cm
below the right costal margin. Laboratory studies show:
Blood
Hematocrit 13%
Hemoglobin 3 g/dL
Mean corpuscular volume 48 ?m3
Platelet count 400,000/mm3
Reticulocyte count 0.8% (N=0.5-1.5% of red cells)
WBC 12,000/mm3
Following appropriate evaluation, which of the following is the most appropriate treatment?
(A) Administration of oral folate therapy
(B) Administration of parenteral iron therapy
(C) Administration of parenteral vitamin B12 (cyanocobalamin) therapy
(D) Transfusion with packed erythrocytes
(E) Transfusion with whole blood
55
Items 55-56
A 75-year-old white woman returns to the office after 6 months of missed appointments. She says she is feeling depressed. You
have been treating her for years for a variety of disorders, including bipolar disorder, hypothyroidism, atrial fibrillation, peptic ulcer
disease and hypertension. She takes daily lithium, levothyroxine, haloperidol, sertraline, benztropine, digoxin, propranolol,
ranitidine and warfarin. At this time she says, "I'm doing fine except for shakiness in my hands." Vital signs are pulse 78/min with
an irregularly irregular rhythm, and blood pressure 160/95 mm Hg. Physical examination shows she has fine tremor of the hands
when they are extended. She says her mood is "a little depressed," and she has no hallucinations or delusions. On memory testing,
she recalls one of three objects after 2 minutes.
55. Which of the following is the most likely cause of the patient's depression?
(A) Benztropine
(B) Digoxin
(C) Haloperidol
(D) Propranolol
(E) Ranitidine
56. You decide to prescribe hydrochlorothiazide. Of her current medications, the hydrochlorothiazide would most likely cause a
serious drug interaction with which of the following?
(A) Haloperidol
(B) Lithium
(C) Ranitidine
(D) Sertraline
(E) Warfarin
END OF SET
57. A 42-year-old woman with a history of multiple sclerosis comes to the office because she had a sudden loss of vision in the
right eye. She has no history of diplopia. On examination, external ocular movements are normal but funduscopic
examination shows pallor of the optic disk. This patient's condition is most likely due to demyelination of which of the
following?
(A) Medial longitudinal fasciculus
(B) Oculomotor nerve
(C) Optic nerve
(D) Trigeminal nerve
(E) Visual cortex
56
58. A 44-year-old Irish-American woman has had arthritis for 10 years, for which she has seen many physicians. She has used
many medications and devices, including copper bracelets from Mexico given to her by friends. She seeks your help because
for the past several months she has had increasing pain and stiffness in her hands. Her hands now show moderate ulnar
deviation of the fingers and she says her wrists and knees also hurt. She has had increasing fatigue for about 1 month, along
with a weight loss of 1.8 to 2.2 kg . Review of her medical records, which she has brought with her, convinces you
that the initial diagnosis of rheumatoid arthritis is correct. She says, "I had several drop attacks during the past 3 months."
She characterizes these attacks as episodes of weakness and loss of feeling in her legs for several minutes. During one of
these episodes, she became incontinent. She currently takes aspirin about four times a day and an occasional dose of
ibuprofen. On physical examination she has facial plethora and swollen and painful metacarpophalangeal and knee joints,
bilaterally. The rest of the examination is normal. Which of the following is the most likely cause of her "drop attacks?"
(A) Adrenal insufficiency
(B) Anxiety
(C) Atlanto-axial instability
(D) Cardiac arrhythmia
(E) Cerebral ischemia
59. A 3-year-old boy is brought to the office by his father because of a 3-month history of decreased activity, poor appetite,
sporadic vomiting, clumsiness and speech regression. Since his birth his family has lived in an old area of the city where
there is demolition of old buildings. Examination of a peripheral blood smear is likely to show which of the following?
(A) Basophilic stippling of erythrocytes
(B) Degranulation of eosinophils
(C) Diminished numbers of platelets
(D) Howell-Jolly bodies
(E) Macrocytic erythrocytes
60. A 68-year-old woman comes to the office for flexible sigmoidoscopy as part of a yearly screening. A 3-cm polyp is found in
the sigmoid colon and is removed. She returns now to the office, 6 hours later, complaining of left lower quadrant pain,
fever, nausea and vomiting. Vital signs are: temperature 38.1°C ,pulse 110/min, respirations 26/min and blood
pressure 120/60 mm Hg. Abdominal examination discloses bowel sounds, tenderness and guarding in the left lower
quadrant. Rectal examination shows no stool and only tenderness superiorly. Which of the following is the most appropriate
next step?
(A) Obtain an angiogram to rule out intestinal ischemia
(B) Obtain immediate consultation with a surgeon
(C) Pass a soft rubber rectal tube under fluoroscopy
(D) Repeat the flexible sigmoidoscopy in order to evaluate the operative site
(E) Start hydrocortisone, intravenously, to decrease any inflammatory response
61. A 32-year-old man and his 29-year-old wife are being evaluated for infertility. The woman's gynecologist reports that the
woman had a normal anatomic and physiologic evaluation and suggests the need for assessment of potential male factors.
On examination, the man is 188 cm (6 ft 3 in) tall with fair skin and little facial hair. His testicles are small and firm, and he
has mild gynecomastia. No sperm are seen on analysis of his semen. Which of the following tests is most likely to establish
the underlying cause of the infertility?
(A) Karyotype from peripheral leukocytes
(B) Serum estrogen and testosterone concentrations
(C) Serum gonadotropin concentrations (follicle-stimulating hormone and luteinizing hormone)
(D) Serum prolactin concentration
(E) Testicular ultrasonography
57
Items 62-63
A 4-day-old Greek-American neonate is brought to the office because of the development of yellow skin and a rash 1 day after
hospital discharge. She weighed 3400 g at birth and is the product of a normal pregnancy. The mother is now gravida 2, para 2 and she is blood
type A, Rh positive. The neonate is blood type O, Rh positive with a negative direct Coombs test. She had an Apgar score of 8 and 9 at 1 and 5
minutes respectively. The neonate was breast-feeding and was doing well at the time of
discharge. Yesterday, the mother says, the neonate developed about 20 small red spots over her face, trunk and extremities. Today on physical
examination there are many papules that have small vesicles with clear to slightly turbid fluid. Her skin color has become yellow. The mother
says that she continues to feed well. On physical examination the neonate weighs 3250 g There is scleral and skin icterus. No organomegaly or
adenopathy is noted. Studies on the neonate show a serum total bilirubin concentration of 8.7 mg/dL, and concentration of serum conjugated
(direct) bilirubin is 0.7 mg/dL.
62. In addition to scheduling a follow-up visit in 1 week, which of the following is the most appropriate advice to give the mother regarding the
icterus?
(A) Ask her to avoid eating foods containing large quantities of carotene
(B) Begin administering small doses of phenobarbital to the neonate
(C) Discontinue breast-feeding until the jaundice has disappeared
(D) Recommend home phototherapy for the neonate
(E) Recommend no change in child care or feeding of the neonate
63. The appropriate steps are taken. You explain to the mother that the neonate's rash is probably erythema toxicum. Which of the following is
the most appropriate management?
(A) 0.5% hydrocortisone cream applied twice a day
(B) Daily wet-to-dry povidone-iodine (Betadine®) soaks with 1×1 gauze pads on each vesicle
(C) Polymyxin ointment applied twice a day
(D) Routine skin care with soap and water
(E) Scrubbing with entsufon cleanser each day firmly enough to unroof the vesicles
58
A 10-year-old girl, who has been undergoing treatment for chronic juvenile rheumatoid arthritis for the past 3 years, is brought to the office
because of painful swelling of the right knee. She has had three episodes of painless swelling of her left knee and ankle, which have subsided
spontaneously with rest and aspirin therapy. She has used no medications between episodes. On physical examination today there is
pronounced redness and warmth around the right knee, and a large effusion is present. Attempts at active and passive motion cause severe
pain.
64. Which of the following is the most appropriate step at this time?
(A) Joint aspiration
(B) Serum antinuclear antibody titer
(C) Serum rheumatoid factor assay
(D) Technetium 99m scan
(E) X-ray films of the joint
65. Which of the following new symptoms or findings, if present, would best indicate the need for further diagnostic studies?
(A) Decreased viscosity of joint fluid
(B) Diffuse increase in technetium 99m uptake around the knee on bone scan
(C) Positive Gram stain of joint fluid
(D) Positive serum rheumatoid factor test
(E) Soft-tissue swelling seen on x-ray films
Items 66-67
A 38-year-old homemaker and mother of four children (ages 5 to 12 years) has been coming to you for management of tension headaches that
have not improved with trials of several appropriate medications. She has been married to a police officer for the past 6 years. You ask if she
has been under extra stress, and she begins to cry. You notice bruises on her arms. On further
questioning, you learn that her husband hits her whenever he is drunk, which is at least 2 nights per week. She says, "He is nice...a good
husband when he's sober. But when he drinks, oh he's awful! He accuses me of cheating on him. Last night he said he would kill me if I try to
leave." Her husband is also a patient of yours.
68. A 24-year-old woman comes to the office for a gynecologic examination. This is her first visit and she has no complaints. She tells you that
she has not had a Pap smear for several years. Menarche was at age 12 years and she has had normal menstrual cycles since then. She has had
several sexual partners in the past but has been with her current partner in a monogamous relationship for 1 year. She reports that she had a
chlamydial infection that was treated several years ago, but she denies a history of other sexually transmitted diseases. She has never been
pregnant. On physical examination her cervix appears friable with a slight area of ulceration. There are several perineal and vaginal lesions that
appear as small "cauliflower-like" projections. The results of the Pap smear, which return in 1 week, show mild dysplasia (LGSIL). Which of
the following factors in this patient's history most closely correlates with the abnormal finding on Pap smear?
(A) Condylomata acuminata
(B) Condylomata lata
(C) Early age at menarche
(D) History of chlamydia
(E) Nulliparity
69. A 50-year-old woman comes to the office for the first time because of recurrent abdominal pain. Review of her extensive medical chart,
which she has brought with her, discloses that she has a long history of varying physical complaints. Definitive causes for these complaints have
not been found despite extensive diagnostic studies, consultations with many physicians and several surgical explorations. She gives dramatic
and exaggerated descriptions of her present and past symptoms, and she makes conflicting statements about her history. She has been
hospitalized at least 23 times since age 18 years. Which of the following is the most likely diagnosis?
(A) Borderline personality disorder
(B) Conversion disorder
(C) Histrionic personality disorder
(D) Occult medical disorder
(E) Somatization disorder
70. At a routine physical examination, the Pap smear of a 27-year-old woman shows evidence of marked inflammation suggestive of moderate
dysplasia (HGSIL). Her last Pap smear 2 years ago was normal. Pelvic examination today is normal. She has never been pregnant and her
menstrual periods are regular. She has been in a stable relationship with the same man for 3 years and she uses a diaphragm with spermicidal
jelly for contraception. Which of the following is the most appropriate next step?
(A) Advise the patient that her partner should use condoms for contraception and repeat the Pap smear in 3 months
(B) Do colposcopic examination of the cervix after application of 5% acetic acid solution
(C) Do conization of the cervix
(D) Reassure the patient and repeat the Pap smear in 3 months
(E) Treat the patient with metronidazole for 2 weeks and repeat the Pap smear in 3 months
71. A 66-year-old African-American man who has been a patient for several years calls the office to report an episode of apparently bloody
urine. He is instructed to come to the office, where urinalysis confirms gross hematuria without proteinuria or casts. The patient denies any
pain and is anxious for an explanation. Physical examination is normal. Which of the following is the most appropriate next step?
(A) Do a transrectal prostatic biopsy
(B) Prescribe a 1-month course of trimethoprim-sulfamethoxazole
(C) Schedule bilateral renal angiography
(D) Schedule cystoscopy
(E) Schedule infusion of the renal pelvis with silver nitrate
72. A 19-year-old white woman returns to the office 2 months after having a medroxyprogesterone injection for contraception. She is
complaining of nonstop bleeding since her menses 3 weeks ago. She is using eight pads a day. She denies any sexual
activity since she received the injection. She realizes that spotting is a side effect; however, she is anxious about the length of time and the
amount of the bleeding. She tells you, "I can't stand this, Doctor. I want the bleeding to stop now!" Repeat pregnancy test is negative. Which of
the following is the most appropriate treatment option for her bleeding?
(A) Conjugated estrogen therapy for 2 weeks
(B) Dilatation and curettage
(C) Reassurance and counseling
(D) A second injection of medroxyprogesterone
(E) Triphasic oral contraceptive therapy for one cycle
73. A 4-year-old boy is brought to the office because he has become unmanageable at his day-care center. At previous visits he exhibited some
behavior problems to which his mother did not set limits. He constantly interrupted situations, seeking his mother's attention. She now reports
that during the past few months his fighting, refusal to obey the day-care workers and
violations of "time out" have become much worse. He began to attend day-care at 6 weeks of age so that his mother could return to work. His
father works as a house painter and he is alcohol-dependent. The boy has a 6-month-old sister who also attends the same day-care center.
Records show his height and weight are at the 5th percentile, and his growth velocity is normal. There were no complications during the
pregnancy with this child and he has not had any significant medical problems. His physical examination today is normal. Which of the following
is the most likely cause for this child's worsening behavior?
(A) Aggressiveness to compensate for a poor self-image caused by short stature
(B) Attention-deficit/hyperactivity disorder
(C) A reaction to his father's drinking
(D) Reduction in his mother's attention because of his new sibling
(E) A toxic reaction to organic fumes from his father's clothes and work materials
74. A 28-year-old woman of Scandinavian descent comes to the office because of fatigue, weakness and palpitations. She is divorced and lives
with her 4-year-old daughter. Complete evaluation shows that this patient has hyperthyroidism and mild ophthalmopathy caused by Graves
disease. Before initiating therapy, the patient wants to know what she can expect in the
future. In advising her about the prognosis, which of the following is the most accurate statement?
(A) Graves ophthalmopathy will resolve as thyroid hormone secretion is lowered
(B) Malignant degeneration of the thyroid is a common complication
(C) She will not be able to become pregnant
(D) The thyroid will continue to increase in size with any nonsurgical treatment
(E) Untreated patients are at increased risk for cardiac arrhythmias
75.
A 58-year-old man comes to the office because of a lesion on the lip, which is shown. The patient says he has had the lesion for about 9 months.
He has not seen a physician for 5 years and he is in the office today only because, he says, "My wife made me come." On physical examination
the lower lip is fixed to the anterior aspect of the mandible. Which of the
following is the most likely diagnosis?
(A) Basal cell carcinoma
(B) Keratoacanthoma
(C) Leukoplakia
(D) Melanoma
(E) Squamous cell carcinoma
76. An 11-year-old girl is brought to the office because of pain in her left calf that she first noted 4 weeks ago and that has gradually increased.
Before the onset of symptoms she was running 10 to 12 miles per week; now she is unable to run because of the leg pain. There is no history of
acute injury despite her intensive training schedule. She is a sprinter who has
won a local qualifying event for a national competition. There is tenderness to palpation over the proximal portion of the posterior calf
musculature. X-ray films are shown. Which of the following is the most likely diagnosis?
(A) Benign neoplasm of bone
(B) Malignant neoplasm of bone
(C) Metabolic bone disease
(D) Osteomyelitis
(E) Stress fracture
77. A 68-year-old man with documented alcohol abuse returns to the office because of abdominal pain and bloating. When you
meet with him, he appears dejected and his eye contact is poor. Physical examination is normal. Since his last visit, he has
moved from the neighborhood where he had lived for 40 years. In addition, he mentions that approximately 6 months ago
breast cancer was diagnosed in his wife; she is currently receiving radiation therapy for bony metastases. The most important
next step in management of his symptoms is evaluation for which of the following?
(A) Delirium tremens
(B) Gastrointestinal bleeding
(C) Pancreatic carcinoma
(D) Situational anxiety disorder
(E) Suicidal ideation
78. A 25-year-old woman who is pregnant with her third child comes to the office for a regular prenatal visit. Medical history
shows that she developed deep vein thrombosis of the left calf in the 22nd week of her last pregnancy 2 years ago. She is now
26 weeks pregnant, and she complains of left calf tenderness during the examination. Deep vein thrombosis is confirmed by
Doppler ultrasonography. Which of the following is the most appropriate management?
(A) Administer intravenous heparin initially, followed by warfarin until delivery
(B) Do venous ligation proximal to the point of obstruction
(C) Prescribe heparin until delivery
(D) Prescribe warfarin until delivery
(E) Prescribe heparin and indomethacin therapy until delivery
79. A 69-year-old Chinese-American man with diabetes mellitus had a myocardial infarction 2 years ago. He has had
exertional angina since then and has been taking propranolol. During the past few days he has had one episode of chest pain
at rest, two episodes postprandially and one at night. Electrocardiogram reveals an old myocardial infarction. Which of the
following is the most appropriate management?
(A) Admit him immediately for cardiac monitoring and adjustment of therapy
(B) Admit him immediately for coronary artery bypass surgery
(C) Decrease the dosage of propranolol and add nitrates and salicylates
(D) Increase the dosage of propranolol and have him return in 1 week
(E) Advise resting from work and sedation at night and digitalization
Items 80-82
You care for a family that consists of a 43-year-old husband, a 42-year-old wife, a 15-year-old daughter and a 12-year-old son.
Each family member is healthy. The 77-year-old maternal grandmother lived with the family until 4 weeks ago when she died
suddenly after a prolonged respiratory illness. Autopsy revealed that she had active pulmonary tuberculosis at the time of her death.
The organism tested sensitive to all commonly used anti-tuberculosis drugs.
80. In following up on the grandmother's illness, which of the following is the most appropriate first step in managing this
family?
(A) Obtain leukocyte count and erythrocyte sedimentation rate on all family members
(B) Obtain sputum or gastric washings of all family members for culture for acid-fast bacilli
(C) Place PPD skin tests on all family members
(D) Place TB tine tests on the two children and PPD skin tests on the adults
(E) Schedule bronchoscopy and alveolar lavage on the adults
81. Complete work-up of each family member reveals no evidence of tuberculosis. The most appropriate next step in
management is to prescribe which of the following?
(A) Isoniazid and rifampin for all family members
(B) Isoniazid and rifampin for the adults only
(C) Isoniazid for the children
(D) Rifampin for the adults
(E) No medications for any family member
The father mentions that his mother-in-law spent many hours with a nephew in the weeks prior to her death. The nephew is 26
years old, has lymphoma and recently completed a course of chemotherapy. You agree to see the nephew, and as part of a thorough
examination, you place a PPD skin test.
82. The nephew's PPD skin test is positive. Complete evaluation of the nephew reveals no evidence of active tuberculosis. Which
of the following is the most appropriate pharmacotherapy at this time?
(A) Isoniazid
(B) Isoniazid and rifampin
(C) Isoniazid, rifampin and ethambutol
(D) Isoniazid, rifampin and streptomycin
(E) Rifampin, ethambutol and pyrazinamide
END OF SET
65
Items 83-84
A 10-year-old white girl is brought to the office for her yearly physical examination. According to her mother the girl has been
teased regularly by other children because she is overweight. Both parents are obese. The girl's growth chart is shown.
83. In reviewing the girl's history you recognize that her weight places her at increased risk for which of the following?
(A) Cushing syndrome
(B) Delayed menarche
(C) Hypothyroidism
(D) Nocturnal enuresis
(E) Slipped capital femoral epiphysis
84. In reviewing the management options for this patient, which of the following is the most appropriate next step?
(A) Give her a written diet to follow
(B) Recommend a behavior-oriented treatment program
(C) Refer her to a commercial weight-loss center for diet management
(D) Refer her to an endocrinologist for hormonal assessment
(E) Tell her not to worry because she will "grow into her weight"
END OF SET
66
Items 85-86
A 27-year-old man comes to the office because of a 1-week history of right knee pain. He says he jogs 3 miles a day and that the
pain in his knee worsens during his run. On physical examination his gait appears to be normal. Examination of the right knee
reveals tenderness and fullness over the medial collateral ligament.
85. Which of the following is the most likely additional finding on physical examination?
(A) Increased anterior laxity with anterior pressure on the tibia
(B) Increased posterior laxity with posterior pressure on the tibia
(C) Pain on compression of the patella against the femur
(D) Pain during internal and external rotation of the tibia while compressing it against the femur
(E) A painful clicking sensation with inward rotation of the foot and extension of the knee
86. The most accurate statement concerning this patient's condition is that he will need which of the following?
(A) A different type of activity for his exercise program
(B) Initial treatment with rest, ice packs and isometric exercises
(C) Referral for an orthotic device
(D) Referral for arthroscopy
(E) Treatment with knee immobilization and crutches
END OF SET
87. A 38-year-old white letter carrier returns to the office for follow-up of an abnormal liver chemistry profile ordered 3 weeks
ago during a routine examination. At that time, his physical examination was normal, but he had a serum AST
concentration of 72 U/L. His serum bilirubin and alkaline phosphatase concentrations were normal. History includes an
episode of hepatitis A at age 22 years. He has no history of transfusions or intravenous drug use. He drinks two to three beers
daily. Today's follow-up test results show:
Serum
Anti-HAV Positive
Anti-HBs Negative
HBsAg Positive
HBeAg Positive
Which of the following is the most appropriate next step?
(A) Begin interferon-alfa therapy
(B) Begin corticosteroid therapy
(C) Have him cease all alcohol consumption and retest him in 2 months
(D) Order hepatitis B virus DNA polymerase study
(E) Schedule liver biopsy
88. A 6-month-old Latino infant is brought to the office by his parents because of intermittent swelling in his right scrotum.
They say the swelling is more pronounced when he cries. The swelling has never been red or "stuck." A right inguinal
hernia is confirmed on physical examination. In discussing repair of the hernia with the parents, you should inform them
which of the following?
(A) Herniorrhaphy can be postponed until age 2 years because many hernias close spontaneously
(B) Herniorrhaphy can be postponed until age 12 years because oligospermia does not develop before age 12
(C) Herniorrhaphy should be scheduled at the earliest convenient time
(D) Herniorrhaphy should be scheduled as an emergency
(E) There is no need to repair a hernia in childhood unless incarceration occurs
67
Items 89-90
A 35-year-old man who is a new city employee was driving a city-owned car when he was struck from behind by another car. He
experienced immediate neck and shoulder pain and was seen and examined by you in the emergency department. Physical
examination showed slight cervical muscle spasm. Cervical spine and shoulder x-ray films at that time were normal. You
reassured him that no structural damage was evident on the studies. He was treated with nonsteroidal anti-inflammatory drugs
(NSAIDs) and was scheduled for a return visit in the office in 2 weeks. At the return visit, he was slightly improved but said he had
been unable to return to work. Physical examination was normal. You prescribed physical therapy and scheduled a return visit in 4
weeks. Today, at the 4-week return visit, he insists he is not improved and demands everything be done to "find out why I still
hurt." Physical examination including neurologic examination, is normal. Repeat x-ray films of the cervical spine are normal.
89. At this time which of the following is the most appropriate management?
(A) Inform the patient that no additional studies are needed
(B) Order a CT scan of the cervical spine
(C) Order an MRI of the cervical spine
(D) Order studies the patient requests
(E) Refer him to a neurologist
90. The patient returns 1 month later with the complaint of persistent neck pain and of some "funny feeling" and pain in his
right hand. You learn his prior employment required repetitive use of both hands. He insists that he had no problem with his
hands and wrists prior to his accident. Neurologic examination at this time discloses only hypesthesia of the medial aspect of
his right little and ring fingers. Which of the following is the most appropriate next step?
(A) Apologize for not recognizing a structural problem from the outset
(B) Discuss the possibility that he is malingering
(C) Explain that this may be a problem unrelated to the accident
(D) Order cervical myelography with CT scan of the cervical spine
(E) Refer him to a neurosurgeon
END OF SET
91. An 80-year-old African-American woman is brought to the office for the first time by her son because she has signs of
mildly decreasing mental function. She is having increasing trouble reading, writing and watching television. She has mild,
stable angina pectoris and she had an uncomplicated myocardial infarction 8 years ago. Physical examination now is normal
except for corrected visual acuity of 20/200 O.U., which appears to be caused by cataracts. Mini-mental state test score is
29 out of 30. Which of the following is the most correct statement about this patient's condition?
(A) Her daily activities would probably improve if she had cataract extraction with lens implantation
(B) Her diminished mental status is a contraindication for a cataract operation
(C) Her history of cardiac disease and advanced age are contraindications for a cataract operation
(D) Her mental status should be reevaluated in 1 year
(E) You need more information to decide whether she would be helped by a cataract operation
92. An 8-month-old infant with trisomy 21 (Down syndrome) has a grade 2-3/6 systolic ejection murmur heard best at the left
sternal border, but it can be heard all over the precordium. S2 is split normally and is loud. She has had two episodes of
pneumonia in the past 2 months. Which of the following is the most appropriate next step?
(A) Do a PPD skin test
(B) Initiate an immunologic evaluation
(C) Order sweat chloride test
(D) Presume the murmur is functional and schedule follow-up visits
(E) Seek consultation with a cardiologist
68
Items 93-94
A 17-year-old boy is brought to the office by his mother who says that he has been increasingly withdrawn and preoccupied for
several weeks. He has been sleeping poorly and has refused to leave the house for the past week because he believes he is being
followed. When his father insisted he attend school this morning, the patient threatened him with a knife. On examination, the
patient is sweating, has dilated pupils and is obviously fearful.
93. Rapport with this patient can best be established by asking which of the following?
(A) "Are you hearing voices?"
(B) "Do you have a special mission to accomplish?"
(C) "Do you think you are being persecuted?"
(D) "How do you feel others are treating you?"
(E) "Who do you think is following you?"
94. You arrange to admit him to the hospital. Following admission, which of the following is the most appropriate immediate
intervention?
(A) Administer haloperidol
(B) Administer lorazepam, intramuscularly
(C) Arrange a conference with the family
(D) Order CT scan of the head
(E) Order toxicologic screening of the urine
END OF SET
95. A 5-year-old boy is brought to the office by his mother because of recurrence of bed-wetting at night. He has a 3-month-old
sister who is healthy. Physical examination is normal. Urinalysis shows:
Specific gravity 1.010
Glucose Negative
Protein Negative
Microscopic 0-1 WBC/hpf, 0 RBC/hpf
Which of the following is the most important information to share with his parents?
(A) This condition will cease if they reprimand him for deliberately wetting the bed
(B) The condition is self-limiting, and they should take care to lessen the emotional impact on their child
(C) This is a potentially serious problem and could represent chronic inflammation of the kidneys
(D) This may be a precursor of diabetes mellitus (E) This signifies a serious underlying emotional disorder
69
96. A 38-year-old woman who is a single mother of two small children comes to the office saying that she feels "halfway tired all
the time." Her physical examination is normal. Toward the end of the visit she says anxiously, "My children just don't listen.
They make me so angry that I feel out of control sometimes." She pauses. "Yesterday my 7-year-old daughter talked back to
me and I slapped her in the face." Her eyes fill with tears. Which of the following is the most accurate statement concerning
this patient?
(A) She should be reported to child abuse authorities
(B) She was clearly a victim of child abuse herself
(C) She would benefit from antidepressant medication
(D) She would benefit from family counseling
(E) She would exhibit better self-control if she were married
NOTE: THIS IS THE END OF THE OFFICE/HEALTH CENTER BLOCK.
ANY REMAINING TIME MAY BE USED TO CHECK ITEMS IN THIS BLOCK.
70
Block 3: Inpatient Facilities
Items 97-120; Time - 30 minutes
You have general admitting privileges to the hospital, including to the children's and women's services. On occasion you
follow your own patients in the critical care unit. Postoperative patients are usually seen on the regular surgical ward unless
the recovery room is specified. You may also be called to see patients in the psychiatric unit. There is a short-stay unit where
you may see patients undergoing same-day operations or being held for observation. Also, you may visit patients in the
adjacent nursing home/extended care facility and the detoxification unit.
ALL ITEMS REQUIRE SELECTION OF ONE BEST CHOICE.
97. A 67-year-old man is admitted to the hospital because of fever, malaise and weight loss for the past 6 weeks. Night sweats
began 4 weeks ago. His medical records show that he has had a tonsillectomy, adenoidectomy, mitral valve
commissurotomy and sigmoid colon resection for diverticulitis. His temperature has been as high as 38.3°C daily.
Today, vital signs are: temperature is 38.8°C , pulse 100/min and respirations 14/min. The patient appears
chronically ill but is in no acute distress. Other physical findings include a cotton-wool exudate in the right eye, crackling
rales at the lung bases and a moderate blowing grade 2/6 pansystolic murmur. There is moderate left lower quadrant
abdominal tenderness. Leukocyte count is 11,500/mm3 with normal differential and hematocrit is 35%. Urinalysis shows
4 WBC/hpf and 10 RBC/hpf. Chest and abdominal x-ray films are normal except for slight left ventricular enlargement. At
this which of the following is the most likely diagnosis?
(A) Bacterial endocarditis
(B) Diverticulitis
(C) Hodgkin disease
(D) Pyelonephritis
(E) Vivax malaria
98. A 60-year-old man is admitted to the hospital because of acute pancreatitis. Laboratory studies show:
Serum Blood
Amylase 1,000 U/L Hematocrit 42%
Calcium 8.4 mg/dL WBC 14,000/mm3
Urea nitrogen 5 mg/dL
Results of serum liver chemistry profile are normal. After 48 hours of fluid therapy and observation, a poor prognosis would
be indicated by which of the following laboratory studies?
(A) Serum alanine aminotransferase (ALT) of 106 U/L
(B) Serum amylase of 2,000 U/L
(C) Serum bilirubin of 4.2 mg/dL
(D) Serum calcium of 6.6 mg/dL
(E) Serum glucose of 200 mg/dL
71
Items 99-100
A 25-year-old man has been on life support systems for the past 48 hours following blunt head trauma.
99. Which of the following is the most important criterion to declare the patient brain dead and to permit removal of life support
systems?
(A) Bedside EEG showing no electrical activity
(B) Decorticate and decerebrate posturing
(C) Failure to respond to electroconvulsive stimuli
(D) Glasgow coma score of 3 or less
(E) Unenhanced MRI of the brain showing dilated ventricles
100. Brain death cannot be established in this patient if there is the presence of which of the following?
(A) Carotid blood flow
(B) Cremasteric reflexes
(C) Elevated serum aminoglycoside concentrations
(D) Hypothermia
(E) Marked cerebral edema
END OF SET
Items 101-102
A 57-year-old man who manages his own accounting firm has a 5-year history of malignant melanoma that has been treated with
local excision and immunotherapy. He now is admitted to the hospital for evaluation of constant pain in his back and left hip and
an 11 kg weight loss. He and his wife of 35 years are worried that "the cancer may be back." Pelvic and abdominal CT scans
show multiple bony metastases. He tells you, "I just want to die. I can't bear this."
101. Which of the following is the most appropriate initial intervention?
(A) Adjust his analgesic regimen
(B) Arrange for him to be transferred to a psychiatric service
(C) Begin antidepressant medication
(D) Initiate hyperalimentation
(E) Refer him to a cancer patient support group
102. Which of the following symptoms would be most suggestive of a major depressive syndrome in this patient?
(A) Anorexia
(B) Expressions of discouragement
(C) Insomnia
(D) Low energy
(E) Withdrawal from family
END OF SET
72
Items 103-104
Patient Chart
Sex: Female
Current age: 22 years
Sociodemographic information: Married, African-American, graduate student
Medical history: First pregnancy; uncomplicated delivery of 4267-g male infant. Fourth degree laceration of perineum
Family history: Noncontributory
Current medications/drug information: Prenatal vitamins
Today's visit
Height: 168 cm (5 ft 6 in)
Weight: 67 kg
Vital signs:
Temperature 37.2EC (99.0EF) Respirations 18/min
Pulse 68/min Blood pressure 128/86 mm Hg
Physical examination: Uterus palpable to level of umbilicus; vaginal laceration appears without inflammation, swelling or drainage, with
sutures intact. Nurses report normal lochia
Laboratory studies: Hemoglobin on first day post partum, 10.8 g/dL
The patient whose chart is shown is being seen in the maternity ward 24 hours after vaginal delivery and repair of a fourth degree
perineal laceration. She is able to walk to the bathroom and to void without difficulty, but she has not had a bowel movement since
delivery. The patient tells you that she is concerned about her insurance company requirement that she stay in the hospital no
longer than 48 hours post partum. She is worried that she will not be ready to leave tomorrow since she is breast-feeding and wants
more help from the nurses. She also says she is worried about bowel function after the perineal repair and wants to stay until she is
sure it will be normal. She asks if you can extend her stay to 72 hours post partum if she is not ready to leave tomorrow.
103. Which of the following is the most appropriate response to her request?
(A) "I'm sure the insurance company will understand if you need another day, so you take whatever time you need
before you go home."
(B) "Let me decide whether or not it is too soon for you to leave the hospital."
(C) "Let's see how you feel tomorrow and we can discuss the most appropriate time to leave then."
(D) "You concentrate on getting better and leave the insurance company to me."
(E) "Unfortunately I have no control over the insurance company, so you had better plan on leaving tomorrow."
104. Which of the following would be the most important indication for extending her hospital stay beyond 48 hours post
partum?
(A) Abdominal distention and lack of appetite at 48 hours post partum
(B) Lack of bowel movement post partum
(C) Need for nursing assistance with breast-feeding
(D) Palpation of the uterus above the pubic symphysis for more than 48 hours post partum
(E) Persistence of lochia for more than 24 hours post partum
END OF SET
73
105. A 56-year-old Japanese-American woman is scheduled for an abdominal operation. She has hypothyroidism that is
controlled with thyroid replacement medication and will be unable to eat or drink for 4 days following the procedure. She is
concerned about receiving her thyroid medication. She should be advised of which of the following?
(A) Although she will have symptoms of hypothyroidism, she will not be given the medication
(B) Although she will not receive the medication, she will have no adverse effects
(C) The medication will be administered daily while the nasogastric tube is clamped
(D) The medication will be discontinued temporarily before the operation
(E) She will receive the medication intravenously
Items 106-108
A 76-year-old woman is admitted to the hospital following a fall at her home. On physical examination, she is dazed, has no
memory of her fall and is unable to respond to any questions about her health. Her skin turgor is poor, and there is bruising around
her left hip. Vital signs are: temperature 36.1EC (97.0EF), pulse 72/min and regular and blood pressure 140/85 mm Hg. X-ray films
of the pelvis taken upon arrival at the hospital show a fracture of the left femoral neck. She is accompanied by her daughter-in-law,
who knows only a portion of her mother-in-law's past medical history. The patient drinks socially, does not smoke cigarettes and
has been active in senior citizen groups. She is known to have taken butabarbital daily for many years and her daughter-in-law
states that since the patient had a seizure several years ago during attempts to lower the butabarbital dosage, she assumed that the
medication was for epilepsy. A bag of medications found at the patient*s home contains multivitamins, an acetaminophen/codeine
combination and naproxen. Uneventful surgical repair of the femur fracture is done the morning after admission. Postoperative
medication orders are for morphine, cephalothin, phenytoin and warfarin.
106. On the evening after the operation, the patient becomes combative, begins to hallucinate and has a brief, generalized
tonic-clonic seizure. Which of the following is the most likely cause for the change in her behavior?
(A) Barbiturate withdrawal
(B) Cerebral concussion
(C) Inadequate treatment of an underlying epileptic disorder
(D) Potentiation of morphine by phenytoin
(E) Warfarin side effect
The acute episode is appropriately managed and her therapeutic regimen is modified. At the time of transfer to an extended care
facility 6 days later, she is alert and cooperative. Her prothrombin time is stable at 1.3 times control values (INR=2.0). Two days
later, she experiences a transient episode of asymptomatic hematuria.
107. Which of the following is the most appropriate treatment?
(A) Administer fresh frozen plasma
(B) Administer intramuscular vitamin K
(C) Administer parenteral protamine
(D) Decrease the warfarin dosage
(E) Prescribe trimethoprim-sulfamethoxazole, orally
108. Which of the following is the most appropriate medication to relieve her arthritis pain while she continues warfarin therapy?
(A) Acetaminophen
(B) Aspirin
(C) Codeine
(D) Naproxen
(E) Phenylbutazone
END OF SET
74
109. A 50-year-old woman is in the recovery room following an uneventful cholecystectomy. Two hours after the procedure she
begins to have ventricular extra-systoles and her systolic blood pressure falls from 110 mm Hg to 90 mm Hg. Arterial blood
gas values while breathing room air are:
PO2 58 mm Hg
PCO2 52 mm Hg
pH 7.30
These signs are most likely the result of which of the following?
(A) Alveolar hypoventilation
(B) Occult hemorrhage
(C) Primary cardiac irritability and failure
(D) Primary hypoxemia resulting from anesthetic gases
(E) Pulmonary embolus
110. A 20-year-old woman who is 35 weeks pregnant with her first child is admitted to the hospital because of persistent
hypertension and 1+ protein on urinalysis obtained 36 hours ago. She is confined to bed awaiting further diagnostic studies
for preeclampsia. Her blood pressure is now 160/100 mm Hg. She is complaining of headaches, blurred vision and epigastric
pain. At this time it is most appropriate to conclude that this patient has which of the following?
(A) Chronic hypertension and requires antihypertensive therapy
(B) Chronic hypertension, but no antihypertensive therapy is necessary
(C) Mild preeclampsia and should continue bed rest pending further diagnostic studies
(D) Severe preeclampsia and requires immediate medical management and delivery
(E) Severe preeclampsia and should continue bed rest pending further diagnostic studies
111. A 52-year-old woman who has had low back pain for several years is admitted to the hospital because the pain has suddenly
worsened. Her current medications include oxycodone, amitriptyline, perphenazine, fluoxetine and trazodone. On physical
examination, the patient is 10% below her ideal body weight, pupils are constricted and skin turgor is poor. She seems
sluggish and her speech is slow. Results of neurologic examination and x-ray films of the lumbosacral spine are normal. If a
medication is responsible for her mental condition, the medication is most likely to be which of the following?
(A) Amitriptyline
(B) Fluoxetine
(C) Oxycodone
(D) Perphenazine
(E) Trazodone
75
Items 112-113
An obese 10-year-old boy with diabetes mellitus is admitted to the hospital because of severe ketoacidosis and cardiovascular
collapse. Initial management consists of cardiac monitoring and intravenous administration of fluids, electrolytes and insulin.
Because of the difficulty in obtaining satisfactory peripheral venous access, the left femoral vein is catheterized percutaneously. Six
hours later, his mental status is improved, blood pressure is 120/70 mm Hg and serum glucose concentration is 250 mg/dL.
Physical examination at that time shows a cold left foot with diminished pulses compared with those of the right foot.
112. Which of the following is the most likely explanation for this patient's diminished pulses?
(A) Paradoxical embolus from the femoral vein
(B) Diabetic arteriopathy
(C) Inadvertent injury of the femoral artery
(D) Intense arteriolar constriction induced by hyperosmolality
(E) Thrombosis of the catheterized femoral vein
113. A deficit of which of the following electrolytes in this patient is most likely to cause death?
(A) Bicarbonate
(B) Calcium
(C) Chloride
(D) Potassium
(E) Sodium
END OF SET
114. A 22-year-old woman who has a 5-year history of ulcerative colitis is admitted to the hospital because of increasing
abdominal pain, diarrhea and hematochezia. Her disease is limited to the descending colon. Current therapy includes
sulfasalazine and corticosteroid enemas. While she is receiving parenteral corticosteroid therapy, she is discovered to be 8
weeks pregnant. Which of the following statements should be emphasized in discussing this pregnancy with her?
(A) Abortion should be considered since ulcerative colitis increases the incidence of premature births and congenital
anomalies
(B) Abortion should be considered because of the adverse effect of pregnancy on ulcerative colitis
(C) Abortion should be considered because of the teratogenic effects of her colitis therapy
(D) Occurrence of spontaneous abortions does not correlate with disease activity
(E) Sulfasalazine and corticosteroids are not known to be teratogenic
76
Items 115-116
You plan to discharge an 81-year-old woman in a few days following a 3-week stay in the hospital for repair of a fractured hip that
she sustained while gardening. She now ambulates with difficulty using a walker, but she is determined to become independent
again and to return to her own home. Her daughter, who is in the room with the patient, says, "I want to take Mother home with me
because I'm concerned that she could fall and break her hip again. Mom says she doesn't really want to leave her own home, but she
will do what is best." The daughter turns to her mother and says firmly, "Isn't that right, Mom?" The mother says, "Yes, I guess so,"
averting eye contact with both her daughter and you by looking down toward the floor.
115. Which of the following is the most appropriate response to the mother?
(A) "Are you sure you want to go home with your daughter?"
(B) "How would you feel if you fell again and had another fracture?"
(C) "Is this really your decision or is it your daughter's?"
(D) "I would like to talk with you in private now."
(E) "You are lucky to have a daughter who wants to take care of you."
116. Which of the following is most likely to prevent another fall and a possible fracture in this patient?
(A) Advise her to walk only when accompanied by an adult
(B) Ensure that she does not have orthostatic hypotension
(C) Provide her with assistance for activities of daily living
(D) Provide her with an electric wheelchair
(E) Request a visiting nurse to assess the safety of her living environment
END OF SET
77
117.
A 57-year-old woman is admitted to the hospital for evaluation of nausea, vomiting, crampy abdominal pain and abdominal
distention. Her medical history includes cholecystectomy and appendectomy 5 years ago. X-ray film of the abdomen is
shown. Which of the following is the most likely cause of her symptoms?
(A) Adhesive band
(B) Femoral hernia
(C) Gallstone ileus
(D) Perforated diverticulum
(E) Sigmoid colon carcinoma
118. A 56-year-old man is admitted to the hospital from the emergency department because of an acute myocardial infarction. He
has no identified risk factors and no history of heart disease. He had no major complications and is discharged after 8 days.
At discharge, which of the following diagnostic tests should be scheduled for the 2-week follow-up visit in order to predict
most accurately his risk for another infarction?
(A) 24-Hour ambulatory electrocardiographic monitoring
(B) Electrocardiogram
(C) Exercise stress test
(D) Radionuclide ventriculography
(E) Ultrasonography of the heart
78
119. A 57-year-old woman with metastatic breast cancer is admitted to the hospital for an investigational chemotherapy protocol.
Her cancer has thus far been unresponsive to treatment. At her request, the attending physician writes a "Do Not Resuscitate"
order. Two days later, at 1:30 AM, the patient has suddenly slumped over and the nurse is unable to obtain a pulse or blood
pressure reading. The patient has agonal respirations and then stops breathing. The first physician to arrive at the patient's
bedside should do which of the following?
(A) Call the attending physician
(B) Call the patient's husband
(C) Pronounce the patient dead
(D) Initiate cardiopulmonary resuscitation
(E) Initiate intravenous chemical resuscitation
120. A 25-year-old woman has just given birth to a 3200-g boy with an Apgar score of 9 at both 1 and 5 minutes. The
physical examination of the neonate is normal. The mother is HIV-positive and received no prenatal care. She acquired HIV
infection from her husband who is also HIV-positive. At the time of delivery her rapid plasma reagin test was 1:164 with a
positive fluorescent treponemal antibody test. When you visit the mother to discuss the medical care of her baby and to obtain
more history, you find her in tears. She asks you what the chances are of her baby being infected with HIV. It is most
appropriate to tell her which of the following?
(A) Since she has no symptoms of AIDS, there is a strong likelihood that her baby will not be infected
(B) Having untreated syphilis at delivery increases her baby's risk for being infected by 50%
(C) Her baby has at least a 50% chance of not being infected and will need to be followed closely since it is difficult to
make the diagnosis at birth
(D) If the results of ELISA and Western blot testing of the baby are positive, the baby is definitely infected
(E) Because she acquired her infection through a heterosexual transmission, there is only a 20% risk that her baby
is infected
NOTE: THIS IS THE END OF THE INPATIENT BLOCK.
ANY REMAINING TIME MAY BE USED TO CHECK ITEMS IN THIS BLOCK.
79
Block 4: Emergency Department
Items 121-144; Time - 30 minutes
Most patients in this setting are new to you, but occasionally you arrange to meet there with a known patient who has
telephoned you. Generally, patients encountered here are seeking urgent care. Also available to you are a full range of
social services, including rape crisis intervention, family support, and security assistance backed up by local police.
ALL ITEMS REQUIRE SELECTION OF ONE BEST CHOICE.
121. A 44-year-old woman comes to the emergency department because of chest pain, shortness of breath and fever. On physical
examination she is in mild respiratory distress. Vital signs are: temperature 37.8°C , pulse 100/min, respirations
24/min and blood pressure 100/60 mm Hg. Auscultation of the lungs discloses rhonchi at the right lung base posteriorly. The
chest pain is in the right anterior region and is pleuritic. Chest x-ray film discloses patchy infiltration of the right lower lobe.
The patient is diagnosed as having community-acquired pneumonia, and she is sent home with a prescription for
erythromycin and an analgesic medication. The patient returns 2 days later complaining of new onset of swelling of her right
leg and foot. Noninvasive vascular studies show a proximal deep venous thrombosis in the right leg, and ventilationperfusion
lung scan is interpreted as high probability for a pulmonary embolus. The patient is given anticoagulation therapy
and recovers. The care this patient received initially is best characterized as which of the following?
(A) An error in diagnostic accuracy
(B) Legal negligence
(C) Malpractice
(D) A therapeutic misadventure
(E) Violation of the principle of nonmaleficence
122. A 46-year-old man with Marfan syndrome, aortic insufficiency and mitral regurgitation comes to the emergency department
because of severe substernal chest pain for the past 3 hours. He describes the pain as tearing in quality and radiating to the
neck. One week earlier he experienced similar, but less severe, chest pain and treated himself with aspirin. Which of the
following is the most likely underlying cause for his worsening symptoms?
(A) Acute bacterial endocarditis
(B) Acute myocardial infarction
(C) Dissection of the aorta
(D) Esophageal reflux with spasm
(E) Perforated peptic ulcer
123. A 15-year-old African-American girl comes to the emergency department because, she says, "something is sticking out of
my bottom since I had a bowel movement this morning." She denies previous episodes, although for more than 1 year she
has had occasional difficulty passing stools. She is not in pain at present, but she is afraid to move her bowels for fear that the
problem will get worse. In response to your questions, she tells you that she moved away from home more than a year ago
and her parents contribute nothing to her support. She has a 6-month-old child and lives with a 28-year-old female cousin.
She has never been married and does not work or attend school. She has no other symptoms. In order to follow the correct
procedure for treating a minor, which of the following is the most appropriate step prior to evaluating this patient's rectal
problem?
(A) Accept the girl's consent as sufficient
(B) Obtain a court order permitting evaluation
(C) Obtain the written consent of at least two licensed physicians
(D) Obtain written consent from at least one of her parents
(E) Obtain written consent from her 28-year-old cousin
80
124. A 29-year-old woman comes to the emergency department because of increasingly severe lower quadrant pain and nausea
for the past 2 days. She says that her menstrual period ended 6 days ago. She is sexually active and does not use any
contraception. Temperature is 38.3°C ,orally. On physical examination, there is bilateral lower quadrant
tenderness with rebound and guarding. Pelvic examination shows leukorrhea at the cervical os and the uterus is tender to
palpation. The adnexal areas are tender but no masses are palpable. Which of the following is the most appropriate
diagnostic study?
(A) Cervical culture
(B) Culdocentesis
(C) Laparoscopy
(D) Serum ?-subunit of human chorionic gonadotropin (?-hCG) concentration
(E) Ultrasonography of the pelvis
125. A 26-year-old man is brought to the emergency department by his family because he says that he is being followed by
gangsters and that they are going to kill him. Temperature is 37.8°C , pulse is 110/min and blood pressure is
160/95 mm Hg. His pupils are dilated. The remainder of the physical examination is normal. The family states that he has a
history of drug abuse. Which of the following drugs most likely caused this reaction?
(A) Alcohol
(B) Cocaine
(C) Diazepam
(D) Heroin
(E) Methaqualone
Items 126-127
A 35-year-old woman is seen in the emergency department because of the sudden onset of severe low back pain 12 hours earlier.
The pain began when she bent over to pick up her 2-year-old child. She has been unable to stay in bed because of the need to care
for her child. Low doses of ibuprofen have eased the discomfort slightly. Careful physical examination, including a neurologic
examination, is normal except for evidence of muscle spasm. She believes she has a herniated disc because 2 years ago her father
developed the sudden onset of back pain that required immediate surgery.
126. Which of the following is the most appropriate first step?
(A) Order CT scan of the lumbar spine
(B) Order MRI of the lumbar spine
(C) Order x-ray films of the lumbar spine
(D) Reassure her and treat with a nonsteroidal anti-inflammatory drug (NSAID)
(E) Request consultation with an orthopedic surgeon
127. She returns 3 weeks later because of persistent left lower leg pain and a weak left ankle-jerk reflex. In addition to pain
management, which of the following is the most appropriate step at this time?
(A) AP and lateral x-ray films of the lumbosacral spine
(B) CT scan of the lumbosacral spine
(C) Diskography
(D) MRI of the spine
(E) Observation
END OF SET
81
128. A 9-year-old boy is brought to the emergency department by his father because the boy is slightly lethargic and has labored
breathing. The father, who is a single parent, reports that the boy is "always thirsty" and "urinates a lot." The boy's pulse is
120/min, respirations are 32/min and blood pressure is 110/65 mm Hg. Laboratory studies show:
Serum
Na+ 132 mEq/L
K+ 4.1 mEq/L
Cl! 92 mEq/L
HCO3
! 6.6 mEq/L
Creatinine 1.0 mg/dL
Glucose 850 mg/dL
The boy is treated with intravenous insulin and isotonic saline solution. Several hours later, he is improved and his serum
glucose concentration is 450 mg/dL. Which of the following is the most appropriate next step?
(A) Add glucose to this intravenous solution
(B) Add potassium to this intravenous solution
(C) Add sodium bicarbonate to this intravenous solution
(D) Begin treatment with intermediate-acting insulin
(E) Change the intravenous solution to hypotonic saline solution
129. A 28-year-old woman who is known to be HIV-positive comes to the emergency department because of increasing
headaches, right-sided weakness and disorientation for the past week. A major motor (grand mal) seizure occurs shortly after
admission. On physical examination following the seizure, vital signs are normal. There is no nuchal rigidity. Funduscopic
examination shows papilledema. There is also right hemiparesis and aphasia. Which of the following is the most likely
diagnosis?
(A) Meningioma
(B) Meningococcal meningitis
(C) Neurosyphilis
(D) Toxoplasmosis
(E) Tuberculous meningitis
130. A 38-year-old nurse comes to the emergency department after leaving work early because of a "horrible headache." She has
had a "cold" with sinus congestion for the past week, and yesterday she began taking an over-the-counter combination of
diphenhydramine and pseudoephedrine. She tells you she has a history of "migraines," multiple allergies, premenstrual
syndrome and depression, for which she takes phenelzine (a monoamine oxidase inhibitor). Vital signs are: temperature
37.2°C pulse 90/min, respirations 16/min and blood pressure 210/118 mm Hg. Which of the following is the most
appropriate action?
(A) Administer meperidine, intramuscularly
(B) Administer phentolamine, intravenously
(C) Order CT scan of the head
(D) Order transillumination of the sinuses
(E) Prescribe oral oxycodone and nasal corticosteroids
82
Items 131-132
A 48-year-old Native American construction worker sustained a comminuted fracture of his left tibia and fibula 4 months ago. For
the past 3 months he has been in the rehabilitation unit with his leg fully immobilized. Three hours ago he suddenly developed
chest pain and shortness of breath, and he has just been brought to the emergency department for further evaluation. On
examination he describes an aching discomfort over the right superior anterior chest and the right scapula posteriorly. The family
history is strongly positive for heart disease.
131. In questioning the patient further, an important point in the history would be the relationship of the pain to which of the
following?
(A) Change of position
(B) Deep breathing
(C) Eating
(D) Swallowing
(E) Walking
132. The presence of a right pleural friction rub in this patient would suggest which of the following?
(A) Pericarditis
(B) Pneumonia
(C) Pneumothorax
(D) Pulmonary embolus with infarction
(E) Pulmonary embolus without infarction
END OF SET
133. A 57-year-old man is brought to the emergency department by the rescue squad after he was found lying unconscious in the
street. No other history is available. On physical examination he has a temperature of 40.0EC (104.0EF) and marked nuchal
rigidity. While awaiting the results of a lumbar puncture, which of the following is the most appropriate intravenous
pharmacotherapy?
(A) Ceftriaxone
(B) Ciprofloxacin
(C) Glucocorticoids
(D) Penicillin
(E) Ticarcillin
83
Items 134-136
A 5-year-old boy is brought to the emergency department by his parents because of a swollen pinna. The father says that the boy fell
off his tricycle in a playground. On physical examination the child's left ear is severely edematous and discolored. There is concern
for the child because this is his third emergency department visit in the past 6 months. When discussing these concerns and the
need for further evaluation with the parents, they become angry and want to take the child home now.
134. Which of the following is the most appropriate next step?
(A) Arrange for the child to be admitted for evaluation and protection
(B) Discharge the child to another relative or neighbor if available
(C) Keep the child and initiate foster care placement
(D) Send the child home with his parents, but notify the police of the situation
(E) Send the child home and arrange for a social worker to make a family assessment on a home visit
135. For which of the following reasons should this case be reported to the child protective service agency?
(A) If another licensed physician agrees with the current assessment and documents this in the chart
(B) If there is a pattern of repeated suspicious injury
(C) If there is proof that the parents inflicted the injury
(D) If the injury is judged to be life-threatening
(E) On the basis of the current suspicion alone
136. Even after informing the parents of the decision to report the case to the child protective services, they continue to deny
having hurt the child and are furious. If the diagnosis of child abuse is incorrect, the reporting physician is liable for which of
the following?
(A) Civil charges
(B) Criminal charges
(C) Medical license revocation
(D) State medical society censure
(E) No damages or penalties
END OF SET
137. A 42-year-old Anglo-American man is brought to the emergency department by his same sex partner because of confusion,
diplopia and mild right arm weakness. On examination the patient is somewhat agitated and shows confusion for recent
events. There is decreased pupillary response on the left with some paresis of lateral gaze on the right. Temperature is
38.3°C Peripheral leukocyte count is increased. Which of the following is the most appropriate next step in
evaluation of his neurologic signs and symptoms?
(A) Bilateral carotid arteriography
(B) CT scan of the head
(C) Electroencephalography
(D) Lumbar puncture
(E) Serum test for HIV antibodies
84
138.
A 60-year-old man comes to the emergency department after sustaining facial injuries in a fight. He is mouth breathing,
apparently due to his injuries, but he denies any respiratory problems. He is known to be alcohol- and drug-dependent. He
has smoked one to two packs of cigarettes per day for 35 years. There is dullness to percussion and rales over the right upper
lobe. Chest x-ray film shown is obtained. Which of the following is the most likely cause of the findings on this x-ray film?
(A) Aspergillosis
(B) Carcinoma
(C) Pneumocystosis
(D) Sarcoidosis
(E) Tuberculosis
85
139.
A 38-year-old Hispanic bank executive comes to the emergency department because of the sudden onset of shortness of
breath, light-headedness, diaphoresis, and weakness. He is afebrile. On auscultation of the lungs, bilateral basilar rales are
heard. Electrocardiogram is shown. Which of the following is the most likely diagnosis?
(A) Acute pericarditis
(B) Hyperventilation syndrome
(C) Myocardial infarction
(D) Pulmonary embolism
(E) Spontaneous pneumothorax
140. A 62-year-old man strikes the steering wheel of his car during a low-speed automobile accident. In the emergency
department he is alert and his vital signs are normal. He complains of mild tenderness on sternal compression. Chest x-ray
film shows a widened mediastinum. The most appropriate first step is to order which of the following?
(A) Bronchoscopy
(B) CT scan of the chest with contrast
(C) A MUGA scan of the heart
(D) Thoracentesis
(E) Thoracic ultrasonography
86
141. A 44-year-old African-American construction worker comes to the emergency department because of excruciating left flank
pain that radiates to his left testicle. He describes the pain as coming in waves, and he says, "This is the worst pain I've had
in my life, and that includes closing my thumb in my truck door." On physical examination he is extremely restless and is in
obvious pain. Genitalia are normal. Abdominal examination is normal except for intermittent guarding with spasms of pain.
Plain x-ray film of the abdomen is normal. Urinalysis and urinary sediment are shown:
pH 6.5
Specific gravity 1.025
Glucose negative
Protein negative
Which of the following is the most appropriate diagnostic study?
(A) CT scan of the abdomen
(B) CT scan of the kidney
(C) Culture of the urine
(D) Determination of serum uric acid concentration
(E) Measurement of 24-hour urinary calcium excretion
142. A 19-year-old woman comes to the emergency department because, she says, "I'm burning up." She is known to staff as an
intravenous drug user. On physical examination a systolic heart murmur is detected over the precordium. An expected
physical finding will be which of the following?
(A) Decreased intensity of S1
(B) Increased intensity of the murmur with deep inspiration
(C) Increased intensity of the murmur with forced expiration
(D) Positive Kussmaul sign (rise in jugular venous pulse with inspiration)
(E) Right-sided gallop
87
143. A 23-year-old man comes to the emergency department because of pain, swelling and erythema over the
metacarpophalangeal joint of the right long finger. Three days ago he struck an opponent in the mouth during an
altercation. On physical examination there is an abrasion over the dorsal surface of the joint, pain on passive motion of the
finger, and exquisite tenderness along the volar aspect of the finger and metacarpal. His temperature is 38.5°C
X-ray films are normal. Which of the following is the most appropriate management?
(A) Splinting for 1 week followed by active range-of-motion exercises
(B) Splinting for 3 weeks followed by active range-of-motion exercises
(C) Splinting, oral antibiotic therapy and reevaluation in 24 hours
(D) Splinting and intravenous antibiotic therapy
(E) Surgical debridement and intravenous antibiotic therapy
144. A 25-year-old man is brought to the emergency department by his wife because he has been vomiting for the past 24 hours.
He has used prochlorperazine suppositories for relief of nausea and vomiting. He now has severe muscle spasms in his neck.
On physical examination there is sustained spasm of the sternocleidomastoid and trapezius muscles with twisting of his head
to the right. Which of the following is the most appropriate pharmacotherapy at this time?
(A) Chlorzoxazone
(B) Dantrolene
(C) Diazepam
(D) Diphenhydramine
(E) Methocarbamol
200000000000800005 Questions!
1 A 92 year old lady from nursing home developed myocardiac infarction and was brought to ED , She told you that she is not for resuscitation if
she dies, What would you do?
A Discuss with patient about the resuscitation and requesting to see family members
B Do an immediate competency test.
C Arrange psychiatric consultation
D Place “No resuscitation “ sing over her bed
E Ask consultant opinion
2 A young woman consult you with her 10 weeks old baby boy, and complains that he is not sleeping well. On examination, the baby looks well
looked after. The mother is anxious , looks unhappy and ignores the baby’s cries. What is the most likely diagnosis?
A Sleep deprivation
B Pueperal psychosis
C Postnatal depression
D Normal behaviours
E GAD
3 Your surgeon college is diagnosed with HIV positive, he is moving interstate to continue his surgical career , and he told you to keep the
secrete . What would you do in this situation? ( repeat question)
A inform to the new employer
B Advise your college to notify the new employer
C Contact medical board and report ASAP..
D Ignore
E Discuss with your supervisor
4 An elderly lady underwent gastric surgery and 12 hour post operation, she developed fever, chest pain and is fearful and accused the nursing
staffs of trying to kill her and wants to leave the hospital . What is the likely diagnosis
A delirium due to infection
B Alcohol withdraw
C delirium due to PE
D Delirium atelectesis
E cardiac infarct
5 a 67 years old smoker who presented with hearing impairment bilaterally, on examination there was normal on Earache tube , normal
Tympanic membrane and External auditory canal. Which the following is most likely the diagnosis?
a)Cholesteatoma
b)Chronic otitis media
c)Acoustic neuroma
d)Herpes
e)Carcinoma tongue
6.A 76 year old man presented with swelling in front of left ear before and after food. Which the following is most likely the diagnosis ?
a)Parotid duct obstruction
b) carcinoma of parotid grand
c ) lymphoma
d) Parotid gland infection
e) All above
7.After thyroidectomy patient developed respiratory distress. What will you do?
a)Remove stitches in the ward
b)Take to theatre and remove stitches
c) Check Calcium level
d) Chest X-ray
e) CTPA to exclude PE
9 A patient presented with loin pain radiating to groin pain with mild haematuria. What is your advice.
a) Do nothing, but follow up
b) Sieve urine for stone
c) Drink lot of water
d) Renal Ultrasound
e) Sent out MSU then start Antibiotics
10 .A lady presented with 3 to 4 ovarian follicular cysts in ultrasound scan, but she has normal FSH,LH and Serum Prolactin level . Which the
following is most likely the diagnosis ?
a)Post menopausal
b)Polycystic ovarian disease
c)Pituitary microadenoma
d)Unknown hypothalamic inhibition
e) all above
11. A 43 year old lady presented to Ed with respiratory distress , you suspicious pulmonary embolism, What is the initial investigation?.
a) Pulmonary angiography (CTPA)
b) V/Q scan
c) D-Dimer
d) Chest X ray
e) Check creatinine level
12.A 23 year old intelligent university student who is only child in family loves to wear mums clothes. He is doing shopping dolls for himself.
Mum is not concerned because this behaviours is only limited to the house. What is this behaviour?
a)Transgender abnormality
b)Hypogonadism
c)Normal variant
d) homosexual
e) Transvestite
13.You are in emergency department and seeing a patient who’s eye open to pain , He can localized the pain and confused, His GCS is ?
a)10
b)11
c)12
d)13
e)14
15 Anaesthetist is unable to intubate due to laryngeal oedema, patient is in resp. distress. What next step would you do ?
a)Tracheostomy
b)Cricothyroidotomy
c)Laryngeal mask ventilation
d) laryngealscope intubation
e) Use introducer
16 A 73 year old man, presented in your GP asking you to write a letter to the housing authorities, because he wants to move from his unit. He
complaints that neighbours always talk about him and probably plotting against him. He is pensioner. Past medical history of dermatitis . What
is your diagnosis (Different types of hallucinations.)
a) audible thoughts
b) persecutory delusions
c) tactile
d) Gustatory hallucination
e) Visual hallucination
17 A 18 year old lady is always Compulsive mirror checking, glancing in reflective doors, windows and other reflective surfaces.
a).Body dysmorphic disorder
b) Munchausen syndrome
c) Delusion
d) Somatization disorder
e) Conversion disorder
18. Which of the following is not a common feature of Anorexia Nervosa?
a) Binge eating.
b). Amennorhea.
c) Self perception of being ‘fat’.
d). Under weight.
e) body dysmophic disorder
19.Motor vehicle accident brought into ED. What is the most appropriate next step?
a) Clear airway
b) Adrenaline
c) Set I/V line
d) GCS
e) Alert the senior doctor
21 A young man presented with gynaecomastia, -What next test would you order?
a) Biopsy
b) Chromosome study
c) LFT’s
d) FSH, LH
e) estrodile level
22 A patient presented with 24 hours right iliac fossa pain. He had similar episodes 3 months ago. However, there is no distension, guarding,
tenderness or rigidity. Which the following is most likely the diagnosis ?
a)Duodenal ulcer perforation
b)Complicated appendicitis
c) carcinoid appendix
d)Diverticulitis
e) Bowel cancer
27.Young boy with pain in buttocks and back of thigh while athletics training.
a)Hamstring origin tear
33.Right posterior lower lobe increased bronchial breathing sounds with dullness on percussion.
a)Lower lobe collapse
b)Consolidation
36.2 year old child with purulent discharge on the meatus and minimally retractable skin with penile swelling.What treatment.
a)Local antifungal
b)Local steroid
c)Local antibiotic
d)Circumscision
1. A 28-year old, nulligravid patient complains of bleeding between her periods and of increasingly heavy menses. Over the past 2 years she has
had two dilation and cruettages (D & C’s) which have failed to resolve her symptoms. Oral contraceptives and antiprostaglandins have not
decreased the abnormal bleeding. It would be appropriate at this time to
A. perform a hysterectomy
B. perform hysteroscopy
C. perform endometrial ablation
D. treat with a GnRH agonist
E. start the patient on a highdose progestational agent
2. All the following statements about danazol are true EXCEPT that
3. Which of the following is true regarding the interview of patients with mental retardation?
6. A 32-year-old mother brings her 7-year-old son to a hospital’s emergency department. The child is found to be restless and displaying
stereotyped hand movements. The mother reports that he was “normal” until 1 year ago and had achieved childhood developmental
milestones at the appropriate times. For the year, the child has shown progressive deterioration in social interactions and communication
abilities and recently has lost control over the bowel training that he had attained earlier. Which of the following is the most appropriate
diagnosis?
A. Rett syndrome
B. Heller syndrome
C. Asperger syndrome
D. Minimal brain damage
7. A 36-year old male patient is admitted to the chronic pain unit for severe chronic back pain. After extensive evaluations, pain disorder
associated with psychological factors and a general medical condition is diagnosed. All of the following are true about pain disorder except
A. 5%
B. 10%
C. 30%
D. 50%
E. 100%
10. A 23-year-old woman and her husband use natura family planning as their contraceptive method. Her menstrual cycle length is variable,
ranging from 26 to 32 days. She does not measure her basal body temperature. The time of her maximum fertility, with the first day of menses
defined as day 1, would be between cycle days
A. 1 and 14
B. 6 and 14
C. 6 and 21
D. 14 and 21
E. 14 and 28
11. You are asked to see a 75-year old retired electrician with an 18-hour history of increasing constant back pain. He tells you that the pin
came on gradually and now goes down into his left groin and thigh. The pain was not precipitated by anything, getting worse and nothing seems
to ease it. Eight months ago he suffered a myocardial infarction. He takes isosorbide dinitrate 10mg Tid for the angina and uses nitrate skin
patches at night. He used to smoke a pack a day, but ceased after his heart attack. He drinks 20gm of alcohol at the weekends. What diagnosis
is most likely?
A. Urinary retention
B. Ureteric colic
C. Acute pancreatitis
D. Leaking of abdominal aortic aneurysm
E. Aortic dissection
F. Musculoskeletal problem
Question 1
Patients suffering from schizophrenia may experience both positive and negative symptoms. All of the following are negative symptoms
EXCEPT:
a) Anhedonia
b) Hallucinations
c) Lack of motivation
d) Emotional blunting
e) Alogia
Question 2
Martha was born at 35 weeks gestation. She required phototherapy for jaundice whilst in hospital. Martha is being breast-fed and is beginning
to gain weight. She is now two months old. Regarding routine childhood vaccination which of the following statements is CORRECT?
a) Vaccinations due to be given at two months of age should be postponed until Martha is at least 13 weeks old to allow for her prematurity.
c) Vaccines for intramuscular injection should be given into the buttock in a premature baby.
* Question 3
Maud is a 70 year old who presents with acute peri-umbilical abdominal pain gradually increasing in intensity. She is vomiting profusely and
develops watery diarrhoea with flecks of blood after an hour of pain. Examination of the abdomen reveals localised periumbilical tenderness
with some rigidity. Rectal examination is normal. An irregular pulse is noted and an ECG is recorded (shown below).
b) Acute pancreatis
d) Biliary colic
* Question 4
A 15 year old young man has sudden onset of severe pain in his right lower abdomen commencing 2 hours ago. He has vomited several times in
the last hour. He is rolling on the bed, stating that the pain is going down into his groin. T 37.1 degrees Celcius, P 110min, BP 135/ 80. Abdomen
- soft, no rebound. Tender right testicle. Your immediate management is:
* Question 5
In a 3 year old child with signs and symptoms suggestive of bacterial meningitis, which of the following is the BEST initial management?
a) Erythromycin IV
b) Gentamicin IV
c) Ceftriaxone IV
d) Phenoxymethylpenicillin oral
e) Amoxycillin oral
Question 6
Atypical antipsychotic (eg. olanzapine) drugs have certain advantages compared to the typical psychotic drugs (eg. Chlorpromazine) in the
treatment of schizophrenia. All of the following are advantages of atypical drugs EXCEPT:
Question 7
Which of the following statements regarding undescended testes is CORRECT?
Question 8
An aspirate in an acutely painful, swollen knee shows the following:
• white cell count 4100/uL (< 2000/uL)
• red blood cells ++
• no crystals
• no organisms cultured
What is your interpretation of these results?
c) Gonococcal arthritis
d) Tuberculosis arthritis
e) Viral arthritis
Question 9
Concerning prostate cancer, which ONE of the following is INCORRECT?
b) The prostate specific antigen (PSA) is elevated in all prostate cancers greater than 1.5 cm
Question 10
Which ONE of the following atypical antipsychotic drugs is optimal as first line therapy in managing the first episode of schizophrenia in a
patient?
a) Risperidone
b) Olanzapine
c) Quetiapine
d) Amuisulpride
e) Clozapine
Question 11
Ara, 5 years old, presents with a painless limp. You suspect Perthes' disorder. Which of the following statements is CORRECT?
c) The white cell count and C reactive protein (CRP) are usually raised
Question 12
Significant protein-energy malnutrition is likely to lead to increased postoperative complications and prolonged recovery times. In assessing a
patient's nutritional status, which ONE of the following findings would be of MOST concern?
d) at the biceps skin fold, the dermis can be felt between finger and thumb
Question 13
In Australia, all of the following are risk factors for suicide, EXCEPT:
a) Schizophrenia
c) Social disadvantage
d) Female
e) Youth
* Question 14
A 15kg child with a known food allergy to peanuts suddenly develops anaphylaxis. The RECOMMENDED immediate management is:
Question 15
Which ONE of the following is a mass population screening test which has been demonstrated to significantly reduce cancer mortality?
Question 16
All of the following groups are at higher risk than average of depression, EXCEPT:
a) Women
b) Postpartum women
d) Urban males
e) Adolescents
Question 17
14 month old Mark is brought in to see you. He has had high fevers and a mild runny nose for three days but has still been quite active. Mark's
temperature has returned to normal today but he has now developed a red maculopapular rash on his trunk. The MOST LIKEY diagnosis is:
a) Measles
b) Chicken pox
c) Erythema infectiosum
d) Rubella
e) Roseola infantum
* Question 18
A 50 year old woman has had major abdominal surgery yesterday. You are called to see her urgently as she has symptoms of shock. Which ONE
of the following examination findings is of MOST concern?
e) BP 80/45 mm Hg
Question 19
Arthur is a 74 year old man who presents reluctantly because his wife feels 'he has not been himself since he came home and is not sleeping
properly.' His appetite is poor and he has lost 6 kg in weight since discharge from hospital eight weeks ago following treatment for myocardial
infarction. He has lost interest in his hobbies and is not taking his medications. He has been a type 2 diabetic for 12 years. On examination he
shows signs of mild cardiac failure. The result of an HbA1C test is 10.5% (target <7%). Which ONE of the following is the MOST likely cause of
Arthur's condition.
a) Senile dementia
b) Uncontrolled diabetes
c) Depression
d) Silent reinfarction
* Question 20
Aidan, a 3 month old boy, presents with paroxysms of coughing associated with cyanosis, lethargy and poor feeding for several days. On
examination, between episodes of coughing, he is afebrile and examination is normal. What is the NEXT step in management?
a) Admission to hospital
Question 21
Simon aged 4 months is diagnosed with pertussis. What is the MOST APPROPRIATE management of Simon's parents?
Question 22
Which of the following conditions is the COMMONEST anxiety disorder encountered in general practice?
b) Phobic disorder
d) Panic disorder
Question 23
Which of the following is NOT a diagnostic feature accompanying anxiety in generalised anxiety disorder (GAD)?
a) Restlessness
b) Fatigue
c) Difficulty concentrating
d) Muscle tension
e) Loss of appetite
Question 24
Martin, aged 50 years, complains of insomnia for two weeks. He says this relates to anxiety about problems at work which surround the end of
the financial year. While he is describing his problem, he is excited and agitated. On examination, his pulse rate is 75/min and BP 135/95 mm
Hg. Which of the following behavioural therapies would be MOST helpful to Martin?
a) Cognitive therapy
d) Relaxation therapy
e) Interpersonal therapy
Question 25
Doris is a 74 year old woman whose husband has died suddenly a week ago. She seeks your help in dealing with insomnia which has been
troubling her since her husband's death. She has tried an over-the-counter preparation which she obtained at the local pharmacy but has not
found it helpful. Which of the following drugs would you offer to prescribe for Doris?
a) Temazepam
b) Zopiclone
c) Zolpidem
d) Amitriptyline
Question 26
Insomnia is defined as inability to:
a) Fall asleep
b) Maintain sleep
Question 27
Miriam, aged 67 years, presents with insomnia for two weeks. She has tried several over-the-counter medications but has not received any
benefit. Further questioning reveals that she is feeling unusually worried about her health, is also irritable and restless and has generalised
muscle tension. You have known Miriam for 5 years since you joined this practice. She has previously complained of 'arthritis' of the hips and
knees, back pain and gastro-oesophageal reflux on various occasions. Which of the following drugs would be MOST suitable for treating
Miriam.
a) Amitriptyline
b) Diazepam
c) Venlafaxine
d) Paroxetine
e) Fluoxetine
Question 28
The prevalence of insomnia in the Australian community is about:
a) 10%
b) 20%
c) 30%
d) 40%
e) 50%
Question 29
Which of the following is a cause of primary insomnia?
c) Behavioural conditioning
e) Bereavement
Question 30
Patients suffering from generalised anxiety disorder (GAD) often go undiagnosed for years because they:
a) Selenium sulphide
b) Zinc pyrithione
c) Betamethasone
d) Miconazole
e) Ketoconazole
Question 32
Amy, a young girl, is worried about a rash on her face which has been there for a few weeks. It is not itchy and has slowly increased in size. On
examination it looks scaly (see figure).
a) Tinea corporis
b) Impetigo
c) Pityriasis versicolor
d) Psoriasis
e) Dermatitis
Question 33
After referral to a paediatric oncologist, Bronwyn is diagnosed as having acute lymphoblastic leukaemia (ALL). Both her parents are very
distressed and visit you to find out more about this condition. Which of the following statements about acute lymphoblastic leukaemia is
INCORRECT?
c) Intrathecal therapy is required only if the disease has spread to the spine
* Question 34
Mary is an attendant at a local accommodation centre and has an intensely itchy rash on her wrists and arms that has been present for the past
few days (see figure).
She has recently bought a new watch and wonders if this is the cause of the problem. The MOST LIKELY diagnosis is:
a) Papular urticaria
b) Tinea
c) Contact dermatitis
d) Eczema
e) Scabies
Question 35
Which of the following statements about routine immunisation is CORRECT?
a) If more than 3 months elapses between triple antigen injections the series should be restarted to obtain adequate immunisation
b) The first triple antigen injection should always be half dose (1/2 cc) to reduce allergic reactions
c) A convulsion following acellular pertussis triple antigen vaccine (DTPa) does not contraindicate its further use
e) Measles vaccine should be given at 6 months of age to protect the child as soon as possible
* Question 36
The MOST appropriate treatment for first degree atrioventricular (AV) heart block is:
a) An artificial pacemaker
c) Atropine
d) Digoxin
e) Requires no treatment
Question 37
Melissa, a 49 year old woman, presents with loss of libido which has been worsening over the past 12 months. She takes a combined oral
contraceptive tablet, and a selective serotonin reuptake inhibitor (SSRI) for depression. She is a non smoker but drinks 1-2 glasses of wine a
night. Which of the following advice would you offer Melissa to help her regain her libido?
Question 39
A 61 year old farmer with a history of hypertension presents with an amnesic episode of sudden onset. When assessed he is perplexed and
bewildered, and repeatedly asks where he is and how he comes to be there. He has short term memory loss but knows his identity and is fully
conscious. He recovers from his memory loss after 6 hours. Neurological examination is normal. His blood pressure is 165/96 sitting. The MOST
LIKELY diagnosis is:
a) Hypertensive encephalopathy
b) Richardson-Steele syndrome
c) Alzheimer's disease
Question 40
Which of the following factors would NOT be a poor prognostic indicator for joint replacement in hip osteoarthritis in a 65 year old man?
a) Age
b) A BMI of 33
c) Associated diabetes
d) Presence of osteoporosis
--
Best Regards
Dr Matthew CAI
Evergo Locum Agency
PO BOX 652
St Merrylands NSW 2160
Info@evergo.com.au
Mobile 0414788973
Tel 02 88127393
Fax 02 98850138